You are on page 1of 59

29

PAGE 29 COMPLETED ON 3 NOVEMENBER

iIF U HAVE PRETHOUGH AN ANSWER AND EVERYTHING MATCHES EXCEPT A FEW WORD U R MISSING A FEW WORDS ....OR NOT TAKING
THE WORDS IN THE RIGHT PERSPECTIVE REREAD

always write the premise conclusion and the resning of the fisrt of the two people talking ....then try n see what is the second attacking by
checking the diagram of the forst person u have drawn ....u will get the answer

when two persons interaction are given see what rae the thouughts said by one
an d if they are countered dierectly by the other though tjey seem so
JUST UNDERSTAND AND SEE IF THEY COUNTER EACH OTHER EXACTLY ........OR SOME OTHER
THINGSI IS SAID ABOUT IT

Owner of JavaJoint: Over the past year, the coffee store has become a daily hang-out for more and more teenagers.
Many of our adult customers do not appear comfortable with this kind of crowd and some of them have told me
that they will no longer stop here for a coffee drink. Since my goal is to maximize our revenue, I want you to
discourage teenagers from coming here and start cul tivat ing a more adul t crowd.
Store manager: Are you sure? On average, each teenager spends just as much as the average adult does, and we
have far more new customers than we have lost over the past year. The store manager responds to the owner by
_____
questioning the veracity of owners evidence
arguing that it would be difficult to implement the owners directive
offering new evidence implying that the status quo is not incompatible with the owners goal
demonstrating that the average teenage customer is as profitable as the average adult customer
offering new evidence refuting that presented by the owner
5.
The owner of the coffee store makes three statements citing evidence supporting
his belief that things are not going well at his store: 1) the store has become a
hangout for teenagers; 2) many of his adult customers do not appear
comfortable; and 3) some of them have told him they will no longer frequent the
store. He then states that his goal is to maximize his revenue and directs his
store manager to discourage the teenagers from frequenting his store and to
cultivate a more adult clientele. We can reasonably infer that the owner believes
that his directive will help him achieve his goal of maximum revenue.
In response, the store manager makes no attempt to refute the managers three
statements, but instead offers some new evidence which implies that following
the owners directive may work against the owners goal of maximum revenue,
further implying that the status quo may be indeed be compatible with that goal.
(A) The store manager makes no attempt to question the veracity of the
statements of the owner. When the manager asks Are you sure? he is
questioning the owners directive (or its effectiveness), not the veracity of the
owners evidence.
(B) The implementation of the owners directive is not mentioned by the manager
and is not relevant.
(C) CORRECT. By asserting that the average teenager spends just as much
money as the average adult and that there are more new customers than former
customers, the manager is presenting new evidence that implies that the current
situation is actually helping the owner achieve his goal more than his directive
would.
(D) The manager only states that the average teenager spends as much as the
average adult; hence, he makes a claim as to the relative revenue generated by
the average member of each group. He does not make any claim as to the
relative profitability of the average member of each group.
(E) The new evidence that the manager presents neither contradicts nor refutes
any of the evidence that the owner had previous presented.

6. Garbage in this neighborhood probably will not be collected until Thursday this week. Garbage is usually collected
here on Wednesdays, and the garbage collectors in this city are extremely reliable. However, Monday was a public
holiday, and after a public holiday that falls on a Monday, garbage throughout the city is supposed to be collected
one day later than usual. The argument proceeds by
treating several pieces of irrelevant evidence as though they provide support for the conclusion
indirectly establishing that one thing is likely to occur by directly ruling out all of the alternative possibilities
providing information that allows application of a general rule to a specific case
generalizing about all actions of a certain kind on the basis of a description of one such action
treating something that is probable as though it were inevitable
6.
We begin by analyzing the structure of the problem:
Premise: Garbage is
usually collected here on Wednesdays, and the garbage collectors in this city
are extremely reliable.

Premise: Monday was a public holiday.

Premise:
After a public holiday that falls on a Monday, garbage throughout the city is
supposed to be collected one day later than usual.

Conclusion: Garbage in
this neighborhood probably will not be collected until Thursday this week.

The
argument is sound and the conclusion seems reasonable.

The language in
the conclusion is not absolute (probably), and this is justified since the
language used in the argumentusually and supposed to beis also
probabilistic. Knowing that the argument is valid, the question you must ask
yourself is, How would I describe the structure of this argument? Answer
choice (A): This answer forces you to make an assessment of the premises
(the evidence) as they relate to the conclusion.

Are the premises irrelevant


to the conclusion? Clearly not. Therefore, this answer is incorrect. Answer
choice (B): This is a half-right, half-wrong answer. The argument does
establish that one thing is likely to occur. But, is this established by ruling
out all of the alternative possibilities? No, to do that would mean presenting
arguments against the garbage being collected on Friday, Saturday, Sunday,
etc. Since this section of the answer choice does not occur, this answer is
incorrect. Also, because the argument does not rule out all the alternatives,
the conclusion is not established indirectly. Answer choice (C): This is the
correct answer. Consider each piece of the argument: providing
informationa variety of information about the garbage situation is provided.
application of a general rulethe general rule is that After a public holiday
that falls on a Monday, garbage throughout the city is supposed to be
collected one day later than usual. to a specific casethe specific case is
the pickup of garbage this week in this neighborhood. Given that all elements
occurred and the answer presents an accurate description of the way the
author made his or her argument, this answer is correct. Now, take a moment
and compare this answer to the prephrase you made after reading the
stimulus. How similar are the two? Given that you may not be familiar with
the language used by the test makers, the two may not be very similar. As
your preparation continues, you will become more comfortable with the
language and your Method of Reasoning pre-phrasing will improve. For
example, note the use in this answer of general rule to describe the last
sentence of the stimulus. The test makers could also have used a phrase like
basic principle to achieve the same result. Your job is to match their
language to what occurred in the stimulus. Answer choice (D): This answer is
an overgeneralizationa situation where one instance is used to make a
broad based conclusion. This is a Reverse Answer since the stimulus
actually uses a general principle and applies it to one instance. In addition,
the language in the answer is far too strong in saying all actions of a certain
kind when the language in the stimulus was probabilistic. Answer choice (E):
This is an Exaggerated Answer. The conclusion states that Garbage in this
neighborhood probably will not be collected until Thursday this week and the
use of probably is a clear and obvious indication that the author does not
think the Thursday garbage pickup is inevitable.

3. The Doppler effect refers to the perceived change in pitch that occurs when the source of a sound is in motion
relative to the observer. For example, the siren on a passing police car will sound higher than its true pitch as the
car approaches, sound the same as its true pitch as the car passes, and sound lower than its true pitch as the car
travels away from the observer. If two trains pass each other going opposite directions on parallel east-west
tracks, which of the following observations provides another illustration of the effect described above?
If the eastbound train blows its horn as they pass, passengers on the westbound train will hear a sound that
decreases in pitch.
If the eastbound train blows its horn as they pass, passengers on the westbound train will hear a sound that
increases in pitch.
If the eastbound train blows its horn as they pass, passengers on the eastbound train will hear a sound that
decreases in pitch.
If the eastbound train blows its horn as they pass, passengers on the eastbound train will hear a sound that
increases in pitch.
If the eastbound train blows its horn as they pass, passengers on the eastbound train will hear a sound that is
steady in pitch.

3.
The argument itself provides one example of the Doppler effect: a siren
decreases in perceived pitch as it approaches, passes and then moves away
from an observer. The answer choice that most closely describes a similar
scenario is a correct illustration of the Doppler effect.
(A) CORRECT. The passengers on the westbound train see the eastbound train
approaching, passing, then traveling away from them. The sound of the horn,
therefore, will decrease in pitch for the westbound passengers.
(B) The passengers on the westbound train see the eastbound train approaching,
passing, then traveling away from them. The sound of the horn, therefore, will
decrease in pitch for the westbound passengers. This choice, however, says that
the sound will increase in pitch.
(C) Relative to the passengers on the eastbound train, the eastbound train and
its horn are in fact standing still! The Doppler effect only describes an perceived
change in pitch that occurs when the source of a sound is in motion relative to
the observer, a situation that is not illustrated by this choice.
(D) Relative to the passengers on the eastbound train, the eastbound train and
its horn are in fact standing still! The Doppler effect only describes an perceived
change in pitch that occurs when the source of a sound is in motion relative to
the observer, a situation that is not illustrated by this choice.
(E) Relative to the passengers on the eastbound train, the eastbound train and
its horn are in fact standing still! The Doppler effect only describes an perceived
change in pitch that occurs when the source of a sound is in motion relative to
the observer, so while it is true that the eastbound passengers would hear their
trains horn at its true pitch, this situation does not illustrate the Doppler effect.

1. If, in a tennis tournament, a match reaches a fifth-set tiebreak, the lower-ranked player always loses the tiebreak
(and, therefore, the match). If Rafael, the second-ranked player, wins a tournament by beating Roger, the topranked
player, then the match must not have included a fifth-set tiebreak. Which of the following arguments most
closely mimics the reasoning used in the above argument?
If a woman with a family history of twins gets pregnant three times, she will have one set of twins. Jennifer, who
falls into this category, had two sets of twins, so she must not have gotten pregnant exactly three times.
If a salesman sells more product than anyone else in a calendar year, then he will earn an all-expenses-paid
vacation. Joe earned an all-expense-paid vacation, so he must have sold more product than anyone else for the
year.
A newspaper can charge a 50% premium for ads if its circulation surpasses 100,000; if the circulation does not
pass 100,000, therefore, the newspaper cant charge any kind of premium for ads.
If a student is in the top 10% of her class, she will earn a college scholarship. Anna is not in the top 10% of her
class, so she will not earn a scholarship.
All of the players on a football team receive a cash bonus if the team wins the Super Bowl. If quarterback Tom
Brady earned a cash bonus last year, he must have been a member of the winning Super Bowl team.

1.
On a "mimic the argument" question, it's useful to use logic notation to
understand the flow of the argument. In this case, we're told that IF A happens
(a match reaches a fifth-set tiebreak), THEN B will definitely happen (the lowerranked
player loses). Standard logic rules tell us that, when given "If A, then B,"
the only definite conclusion we can draw is "If not B, then not A." In other words,
if A always leads to B, and B doesn't happen, then A can't have happened either.
The second sentence of the argument shows this principle: If not B (the lowerranked
player doesn't lose), then not A (there wasn't a fifth-set tiebreak). So we
need to find another argument that follows this pattern: If A, then B; if not B, then
not A.
(A) CORRECT. If A (a woman with a family history of twins gets pregnant 3
times), then B (she will have 1 set of twins). Note that these numbers are
precise: if she gets pregnant exactly three times, she will have exactly one set of
twins. If not B (a woman with a family history of twins has 2 sets of twins - that is,
not 1), then not A (she must have gotten pregnant either fewer than 3 times or
more than 3 times - that is, not exactly 3 times).
(B) If A (a salesman sells more product than anyone else), then B (he will earn
an all-expenses-paid vacation). If B (Joe earned the trip), then A (he must have
sold more than anyone else). We can see why logic rules do not include "if B,
then A" as a logical conclusion: A may always lead to B, but B does not
necessarily have to lead to A. There may be other ways to earn the trip besides
selling more than anyone else.
(C) If A (a newspaper's circulation surpasses 100,000), then B (the newspaper
can charge a 50% premium). If not A (the circulation doesn't surpass 100,000),
then not C (the newspaper cannot charge any premium). The final assertion
here does not match the initial A / B argument We know nothing about any other
premium the newspaper might charge; we are only given information about
charging a 50% premium.
(D) If A (a student is in the top 10% of the class), then B (she will earn a
scholarship). If not A (Anna is not in the top 10%), then not B (she won't earn a
scholarship). We can see why logic rules do not include "if not A, then not B" as
a logical conclusion: A may always lead to B, but it doesn't have to be the only
way to reach B. There may be other ways to earn a scholarship besides being in
the top 10% of the class.
(E) If A (the team wins the Super Bowl), then B (the players receive a bonus). If
not A (a player was not on the winning team), then not B (the player won't receive
a bonus). We can see why logic rules do not include "if not A, then not B" as a
logical conclusion: A may always lead to B, but it doesn't have to be the only way
to reach B. There may be other ways to earn a bonus besides winning the Super
Bowl.
Machine harvesting Corn - GMAT PREP CR
Sun Aug 09, 2009 5:33 pm

Which of the following most logically completes the argument?

A new machine for harvesting corn will allow rows to be planted only fifteen inches apart, instead of the usual thirty inches. Corn planted this closely
will produce lower yields per plant. Nevertheless, the new machine will allow corn growers to double their profits per acre because __________.

(A) with the closer spacing of the rows, the growing corn plants will quickly form a dense canopy of leaves, which will, by shading
the ground, minimize the need for costly weed control and irrigation
(B) with the closer spacing of the rows, corn plants will be forced to grow taller because of increased competition for sunlight from neighboring corn
plants
(C) with the larger number of plants growing per acre, more fertilizer will be required
(D) with the spacing between rows cut by half, the number of plants grown per acre will almost double
(E) with the closer spacing of the rows, the acreage on which corn is planted will be utilized much more intensively than it was before, requiring more
frequent fallow years in which corn fields are left unplanted

I debated a lot between (A) and (D) on the exam and picked D. But the OA is A. Instructors, can you please tell why D is wrong....I am doing this a lot on
the exam. I always narrow it down to final 2 and end up picking the wrong one. Please help....
jeffzolman
Wed Aug 12, 2009 2:50 pm

D is wrong because it does not take the argument into account and the formula Sales - Costs = Profit...

The argument is "the new machine will allow corn growers to double their profits per acre".

In general, arguments that discuss profits need to be addressed with the formula of Sales - Costs = Profits. Answer choice A attacks the "cost" part of the
formula. If the farmer lowers the costs, profits will rise!

All answer choice D really says is that the number of plants will double. You can not assume that sales will double with costs only rising marginally to
create double the amount of profits. This might not be the best advice but you can assume D from the statement - if the spacing is cut in half, the
number of plants must double.

Hope this helps...


RonPurewal
ManhattanGMAT Staff
jeffzolman wrote:D is wrong because it does not take the argument into account and the formula Sales - Costs = Profit...

The argument is "the new machine will allow corn growers to double their profits per acre".

In general, arguments that discuss profits need to be addressed with the formula of Sales - Costs = Profits. Answer choice A attacks the "cost" part of the
formula. If the farmer lowers the costs, profits will rise!

All answer choice D really says is that the number of plants will double. You can not assume that sales will double with costs only rising marginally to
create double the amount of profits. This might not be the best advice but you can assume D from the statement - if the spacing is cut in half, the
number of plants must double.

Hope this helps...

well done, jeffzolman.

also, note that choice (d) contains something that is actually self-evident from the information that's already in the passage.
in other words, if you cut the spacing between rows by 1/2, then by definition you're going to be able to plant about twice as many plants. therefore, (d)
is simply a repetition of what's already in the argument, and adds virtually nothing.
4.
FACT: Corn planted this closely will produce lower yields per plant.
Result: the new machine will allow corn growers to double their profits
We need to find an option that will adequately explain why despite the yield per acreage going down,the farmers are able to generate a profit.

A. CORRECT. By telling us that the machine will minimize the need for costly weed control and irrigation, this option tells us why the farmers might
make a profit despite lowered yields.

B. Taller plants does not necessarily mean more yield. Also we already know that the yield was less that before. making this option irrelevant.

C. This actually casts doubts on increased profit by indicating that the expense for the farmer might actually go up.

D. Same as B. we already know that the yield was less that before. This makes the option irrelevant.

E. Same as C. By telling us that the number fallow years will increase, this tell us that the farmers may have to wait for longer than before to being
cultivation.

11. Radio stations with radio data system (RDS) technology broadcast special program information that only radios
with an RDS feature can receive. Between 1994 and 1996, the number of RDS radio stations in Verdland
increased from 250 to 600. However, since the number of RDS-equipped radios in Verdland was about the
same in 1996 as in 1994, the number of Verdlanders receiving the special program information probably did not
increase significantly. Which of the following is an assumption on which the argument depends?
FOR ASSUMPTION QUESTION JSUT QUESTION YOURSELF HOW IS IT POSSIBLE ANND U WILL GET THE NASWER

A. Few if any of the RDS radio stations that began broadcasting in Verdland after 1994 broadcast to people with
RDS-equipped radios living in areas not previously reached by RDS stations.
B. In 1996 most Verdlanders who lived within the listening area of an RDS station already had a radio equipped to
receive RDS.
C. Equipping a radio station with RDS technology does not decrease the station's listening area.
D. In 1996 Verdlanders who did not own radios equipped to receive RDS could not receive any programming from
the RDS radio stations that began broadcasting in Verdland after 1994.ALREADY GIVEN IN THE RAGUMENT ............ASSUMPTION ACANNOT BEGIVEN
IN THE ARGUMENT ...THIS HAS NO NEW INFOR
E. The RDS radio stations in Verdland in 1996 did not all offer the same type of programming.

yes.

ASSUMPTIONS must be statements IN ADDITION to the premises given in the passage. they can't merely be repeats of the information that's already
there.

you won't often see this happen, by the way, but, if you get a statement that appears to regurgitate one of the premises, then you should strike it out.
Here's a diagram for this argument:

(# of rds-equipped stations increased 250-->600)

BUT

(apprx same # of rds radios in Vland)

THEREFORE

(apprx same # of people receiving rds signals in Vland)

if you make this diagram, it should be clear that there's a logical leap between the latter two statements: the author is assuming a direct correlation
between the # of rds-equipped radios and the # of people who actually receive rds signals with those radios.

in order to make this connection, you need to assume that nobody, or almost nobody**, with an rds-equipped radio is now (in '96, that is) able to
receive a signal but wasn't able to receive a signal back in '94. that's pretty much what A. says.

B. Irrelevant. As it does not directly discuss whether these people already had a RDS radio before the station was built , it does not fill the gap.

C. C is out of scope. It says that equipping a radio with the RDS feature does not decrease the range of the radio. But who cares? Even if it did, this
would have been the case for 1994 as well as 1996. We need something that connects the data about radios and 94-96 with the claim that THE SAME
NUMBER OF PEOPLE received the programming, even though no one bought new radios.

D. D is not an assumption - it is a fact. We are told that you cannot get this type of signal without the right equipment. An assumption must be
something that is unstated.

E. Lets negate this:

The RDS radio stations in Verdland in 1996 all offered the same type of programming.
This does not affect our conclusion in any way and hence is irrelevant.
rohit21384 wrote:Case -1
since the number of RDS-equipped radio is the same in 1996 as in 1994 , people in this area B would be without RDS-Equipped radio and hence will not
recieve the broadcast of special program infomration. to recieve the broadcast, they will have to buy RDS-equipped radio . So, total number of people
recieving special program information ---100 people in area A of verland and 0 in area B of verland.

this is the problem.

you're assuming that no one in area "b" had an rds-equipped radio prior to 1996.

not only is that a baseless assumption, but it's actually contradicted by the answer choice, which says:
people with RDS-equipped radios living in areas not previously reached by RDS stations.

so you can't assume that there are 0 people in area "b" with the radios.

in fact, this is the way you should do it:


1994:
100 people in area "a" with equipped radios - CAN get signal
100 people in area "b" with equipped radios - CANNOT get signal
total 100 can get signal

1996:
100 people in area "a" with equipped radios - CAN get signal
100 people in area "b" with equipped radios - CAN NOW ALSO get signal
total 200 can get signal

15. Some airlines allegedly reduce fares on certain routes to a level at which they lose money, in order to drive
competitors off those routes. However, this method of eliminating competition cannot be profitable in the long
run. Once an airline successfully implements this method, any attempt to recoup the earlier losses by charging
high fares on that route for an extended period would only provide competitors with a better opportunity to
undercut the airline's fares. Which of the following, if true, most seriously weakens the argument?

STRUCTURE IS ...........FACT ............CONCLUSION..............THEN REASONING FOR IT


CONSLUION NEGATED ...AND U HAVE TO FIND A WAY TO GET AWAY FROM THE REASONING GIVEN TO COUNTER THE CONLUSION

A. In some countries it is not illegal for a company to drive away competitors by selling a product below cost.
B. Airline executives generally believe that a company that once underpriced its fares to drive away competitors is
very likely to do so again if new competitors emerge.
C. As part of promotions designed to attract new customers, airlines sometimes reduce their ticket prices to below
an economically sustainable level.
D. On deciding to stop serving particular routes, most airlines shift resources to other routes rather than reduce
the size of their operations.
E. When airlines dramatically reduce their fares on a particular route, the total number of air passengers on that
route increases greatly. 15.
the argument rests on the premise that, once the leading airline raises its prices back up to 'normal' higher levels, other airlines will jump right back
into the fray.

A. IRRELEVANT. This is an exception. We are not discussing such countries here.

B. CORRECT. it states that other airlines are likely to continue to stay away, even after the big mean price-cutting airline raises its prices back up. (if
their executives believe that 'big air' will simply lower its prices again if they try to wedge back into the market, then they'll stay out.)

C. IRRELEVANT. We already know what strategy the airlines plan to adopt.


D. IRRELEVANT. Because the passage and its conclusion aren't at all affected by what the other airlines do if they decide to stop serving some
particular route. all that matters is that they decide to stop serving the route; the subsequent decisions are immaterial.

E. IRRELEVANT. As the argument doesn't turn on what happens during the low-fare period (it's most important premises concern what happens after
prices are raised back)

As with all arguments, I like to first start by reading the question and then breaking down the argument into conclusion and premises. First the
question:

Shawshank wrote:

The argument is vulnerable to criticism on the grounds that it gives reason to believe that it is likely that

Alright so I know now that I will be dealing with weaknesses in the argument. So I will keep that in mind as I break it down.

Shawshank wrote:

At present the Hollywood Restaurant has only standard-height tables. However, many customers come to watch the celebrities who frequent the

Hollywood, and they would prefer tall tables with stools because such seating would afford a better view of the celebrities. Moreover, diners

seated on stools typically do not stay as long as diners seated at standard-height tables. Therefore, if the Hollywood replaced some of its seating

with high tables and stools, its profits would increase.

Conclusion: Replace seating and profits will go up

Premise: People go to restaurant to see celebs


Premise: People want tall tables and seats to see the celebs
Premise: Diners on stools don't stay as long

Alright so there is the argument. Not a lot there to support the idea that profits will go up expect for the fact that there might be a faster turn over
of tables. But there are a lot of assumptions here:

1. tall stools and tables won't deter people from spending as much as they did with normal tables
2. People actually want to see celebs and not eat food
3. Celebs will continue to come even if it easier for people to see them at the taller tables
...

Now it is time to look at the answer choices and see what makes the argument vulnerable. We need to look for reasons for why profits might not
increase.

Shawshank wrote:

(A) some celebrities come to the Hollywood to be seen, and so might choose to sit at the tall tables if they were available

Well, this is not a problem. This is just more support for having taller tables. The celebs will come to sit at the tall tables making it easier for
people to see them. This is not a criticism. Eliminate.

Shawshank wrote:

(B) the price of meals ordered by celebrities( TRAP...BEWARE OF WORDING WHILE READING ) dining at the Hollywood compensates for

the longer time, if any, they spend lingering over their meals

This too focused on the celebrities. The argument and the restaurant does not base its profits on how much celebs spend. Profits are based on all
the other people coming to the restaurant. This is too narrowly focused so eliminate.

Shawshank wrote:

(C) a customer of the Hollywood who would choose to sit at a tall table would be an exception to the generalization about lingering
This gets at one of the assumptions I had. If someone sits at a tall table, will they stay longer or leave faster. Here we have a possible example, or
a question, about what these customers are like. The argument assumed that people at the Hollywood would leave quickly when at a tall table, like
at other restaurants. But what if having a tall table means you can see the celebs. This might be a reason to stay. And thus there would not be a
fast turn over. People might stay longer because they have a good view of a celeb whereas before, without a good view, people would just eat and
leave. This looks like the answer.

Shawshank wrote:

(D) a restaurant's customers who spend less time at their meals typically order less expensive meals than those who remain at their meals longer

This is also close to what I was saying in the assumptions. But the problem is that this does not necessarily weaken the argument. People who stay
less time order less expensive food which might cut into the profits. But if you have more people coming in to eat, and you can sit more people
during your business hours, then ordering less expensive food won't be a problem. So this might be a problem, but not necessarily. Answer choice
(C) would necessarily weaken the argument and cut into profits always. So this answer is not as good as D. But is a good tempting choice.

Shawshank wrote:

(E) with enough tall tables to accommodate all the Hollywood's customers interested in such seating, there would be no view except of other tall

tables

This might also be a problem, but the argument doesn't say that they are going to jam tables into the restaurant. There is no mention of adding
more seating. They are merely going to replace tables that they have. So this is outside the scope of the argument and wrong.

I hope that I was able to shed some light on this question.

A and B are straight out as they discuss about the celebrities coming for dinner.

I find E somewhat irrelevant to the argument at hand.

D- This is a pretty general statement regarding people staying at the tables longer and ordering expensive food.
no where does it bring out the difference between standard height and tall tables.

C- Bingo! The general trend what people follow is that they come to holly wood only to watch celebrities and just linger on ;
they plan to replace standard height with tall tables so that they can increase their profits, but what if people are just lingering ?
How will that increase the profits?

Am I right with my understanding?

20. Lets first find out which of the two bold-face statements( if any) form the conclusion.
the first bold-face is followed immediately by therefore, xxxxx, so we know at once that it is not the main conclusion of the argument (since it's
being used to justify something else).

The second bold-face on the other hand, is used to establish a position by indicating what maybe the RESULT of the action taken by the Pro-Tect
company.

Note:

Cause:Pro-Tect Insurance Company's total payout on car-theft claims has been larger than the

company can afford to sustain.Pro-Tect cannot reduce the number of car-theft policies it carries, so it cannot protect itself against continued large
payouts that way

Effect:Therefore, Pro-Tect has decided to


offer a discount to holders of car-theft policies whose cars have antitheft devices. Many policyholders will

respond to the discount by installing such devices, since the amount of the discount will within two years

typically more than cover the cost of installation.

Result:Thus, because cars with antitheft devices are rarely stolen, Pro-Tect's plan is likely to reduce its annual payouts.

Now lets look at the options that indicate the second bold-face to be the conculsion.

A) indicates both the bold-faces are conclusions. INCORRECT.

B) CORRECT. Aptly suggests that bold-face 1 is the problem and bold-face the conclusion to that problem

C) Indicates boldface 1 to be the conclusion.

D) the second bold-face is not a prediction but the conclusion. The prediction in the para is Many policyholders will respond to the discount by
installing such devices, since the amount of the discount will within two years

typically more than cover the cost of installation

E) indicates that neither of the bold-faces form the conclusion. We know this to be incorrect.

E- The first presents a development whose likely outcome is at issue in the argument;

The development (can't reduce the number of policies) has an outcome (can't protect ourselves against large payouts), but there isn't an issue about this
outcome. If there were, another opinion would have to come in saying "I think we CAN protect ourselves against large payouts, even if we can't reduce
the number of policies".

the second is a judgment the argument uses in support of its conclusion about that outcome.

The second isn't a judgment that is used to support a conclusion. It is the main conclusion- the plan will reduce payouts.
Joe Lucero

During the past year, Pro-Tect Insurance Company's total payout on car-theft claims has been larger than the
company can afford to sustain. Pro-Tect cannot reduce the number of car-theft policies it carries, so
it cannot protect itself against continued large payouts that way. Therefore, Pro-Tect has decided to
offer a discount to holders of car-theft policies whose cars have antitheft devices. Many policyholders will
respond to the discount by installing such devices, since the amount of the discount will within two years
typically more than cover the cost of installation. Thus, because cars with antitheft devices are rarely
stolen, Pro-Tect's plan is likely to reduce its annual payouts.
In the argument above, the two portions in boldface play which of the following roles?
A. The first and the second are both evidence offered by the argument as support for its main conclusion.
B. The first presents a problem a response to which the argument assesses; the second is the judgment reached
by that assessment.
C. The first is the position the argument seeks to establish; the second is a judgment the argument uses to
support that position.
D. The first is a development that the argument seeks to explain; the second is a prediction the argument makes
in support of the explanation it offers.
E. The first presents a development whose likely outcome is at issue in the argument; the second is a judgment
the argument uses in support of its conclusion about that outcome.

I need some explanation in understanding the solution to the following question:

When people evade income taxes by not declaring taxable income, a vicious cycle results. Tax evasion forces lawmakers to raise income tax rates,
which causes the tax burden on nonevading taxpayers to become heavier.This, in turn, encourages even more taxpayers to evade income taxes by
hiding taxable income.The vicious cycle described above could not result unless which of the following were true?

(A) An increase in tax rates tends to function as an incentive for taxpayers to try to increase their pretax incomes.
(B) Some methods for detecting tax evaders, and thus recovering some tax revenue lost through evasion,bring in more than they cost, but their
success rate varies from year to year.
(C) When lawmakers establish income tax rates in order to generate a certain level of revenue, they do not allow adequately for revenue that will be
lost through evasion.
(D) No one who routinely hides some taxable income can be induced by a lowering of tax rates to stop hiding such income unless fi nes for evaders are
raised at the same time.
(E) Taxpayers do not differ from each other with respect to the rate of taxation that will cause them to evade taxes
The answer: C.
The explanation states ".... It is said that tax evasion forces legislative increases in tax rates to cover the loss of tax revenues...."

It is not mentioned anywhere in the question about loss of tax revenues.


like
0

25th December 2012 07:14:44 AM

Rohit
Premium User

15th June 2012


Joined on
05:39:51 AM
Threads 3
Posts 22
25th March 2014
Last Login
09:36:15 AM
When law makers, in order to generate the adequate revenue, fix the income tax rates based on the number of potential tax payers, they do so
considering the ideal scenario where everyone that is eligible to pay taxes does so...Then, the situation described in the original statement takes place.
But, if had they kept provision for non tax payers while fixing those rates, they would perhaps keep the initial rate higher and that would ensure that
the adequate amount is generated. While it would hurt the tax payers, it wouldn't give birth to a new generation of tax evaders.
Tax revenue is the implied part.
like
0

25th December 2012 07:21:13 AM

Stephen
Premium User

3rd October 2012


Joined on
10:20:27 AM
Threads 22
Posts 90
26th May 2013
Last Login
07:18:18 AM
My Two cents on any posting: [please leave the question open and wait for users like me to answer so that we also arrive at the official answer :) it
creates a good learning experience for all of us, specially ones who are going to try the verbal review at a later time]. That being said, the reason why
I will chose "C" is:

Since the statement is that tax evasion causes a vicious cycle, if the lawmakers know that folks are going to evade anyways (loop holes, lowering tax
bracket, whatever), they should anticipate this lost revenue and include it in the tax bracket, so that the the final revenue that the government or
whatever receives in the end == what they expect for that year. This can possibly eliminate the vicious cycle.

Hope I make sense.

Any other explanations, well appreciated.

Transnational cooperation among corporations is experiencing a modest renaissance among United States
firms, even though projects undertaken by two or more corporations under a collaborative agreement are less
profitable than projects undertaken by a single corporation. The advantage of transnational cooperation is that
such joint international projects may allow United States firms to win foreign contracts that they would not
otherwise be able to win. Which of the following is information provided by the passage?
A. Transnational cooperation involves projects too big for a single corporation to handle.
B. Transnational cooperation results in a pooling of resources leading to high-quality performance.
C. Transnational cooperation has in the past been both more common and less common than it is now among
United States firms.
D. Joint projects between United States and foreign corporations are not profitable enough to be worth
undertaking.
E. Joint projects between United States and foreign corporations benefit only those who commission the projects

67.

In Inference question, we cant bring any outside information. Correct answer is the one which we can deduce from
the passage above.
A. Incorrect. The passage does not talk about what type of projects involved in the Transnational cooperation or who can handle those projects.

B. Incorrect. We know that it is a collaborative agreement without any further details on the agreement

C. CORRECT.
first, the word renaissance (rebirth) implies that transnational cooperation is now on the rise , after previously having a
lower level. so that takes care of the lower in the past part of the correct answer.

second, the renaissance is described as modest. the only way this description would be reasonable is if transnational cooperation
has not re-achieved its previous levels; there's the higher in the past part.

D. Irrelevant.

E. Irrelevant.
Re: Transnational cooperations
Fri Mar 25, 2011 11:36 pm

is this problem really from gmat prep?


i ask because the question prompt is extremely unusual; in effect, the question prompt is basically asking you to regurgitate one of the premises of the
argument.

in any case, whether this question is official or not, it's a decent question. here is the evidence for the given answer choice:

first, the word "renaissance" (rebirth) implies that transnational cooperation is now on the rise, after previously having a lower level. so that takes care
of the "lower in the past" part of the correct answer.

second, the renaissance is described as "modest". the only way this description would be reasonable is if transnational cooperation has not re-achieved
its previous levels; there's the "higher in the past" part.
Pueden hacerle preguntas a Ron en castellano
Potete fare domande a Ron in italiano
On peut poser des questions Ron en franais
Voit esitt kysymyksi Ron:lle mys suomeksi

Un bon vtement, c'est un passeport pour le bonheur.


Yves Saint-Laurent
pm

Re: Transnational cooperations


Sat Sep 17, 2011 3:47 am

but, Transnational cooperation has in the past been both more common and less common than it is now among United States firms
what is the meaning about both common and less common? I realzie that in the past the level may be higher than now( modest) , but how to
understand both common and less common?

Re: Transnational cooperations


Tue Sep 20, 2011 10:13 am

saintjingjing wrote:but, Transnational cooperation has in the past been both more common and less common than it is now among United States firms
what is the meaning about both common and less common? I realzie that in the past the level may be higher than now( modest) , but how to
understand both common and less common?

these 2 things were true at different times.


"renaissance" implies that the level has gone up; "modest" implies that it hasn't gone all the way back up to previous levels.
Pueden hacerle preguntas a Ron en castellano

62.

Court ruled that companies can reject applicant if there's a 90% chance applicant would suffer heart attack from that job.

Justification for the ruling: protected both employees and employers.


Assumptions: the employees and the employers both need protecting. The ruling actually does protect both from something bad.

Question: we could not use this ruling if something else were true. What is that thing? (Note that the question says "the ruling... could not be
effective" not just "might not be as effective" - so the ruling could never be justified if the correct answer were true.)

A. Is it always true that the best interests of employees and employers aren't the same? Not according to this choice - it's often true. So sometimes
the best interests are actually the same. In those cases, the ruling could still be effective. Because the question asked us to find a circumstance that
meant the ruling could not (ever) be effective, this one doesn't work.

B. CORRECT. If this were true, and there was no way of assessing the risk, the ruling could NEVER be effective.

C. Irrelevant. We are not discussing job risks but the predisposed condition in an employee to suffer a heart attack at the time of hiring.

D. to an extent justifies the ruling by indicating that its in the best interest of both the employees and employers.

E. Even if this were true, since such a ruling MAY only cause the number of applicants to decline and not stop all together, such a ruling may still be
effective

62. Recently a court ruled that current law allows companies to reject a job applicant if working in the job would
entail a 90 percent chance that the applicant would suffer a heart attack. The presiding judge justified the
ruling, saying that it protected both employees and employers. The use of this court ruling as part of the
law could not be effective in regulating employment practices if which of the following were true?
A. The best interests of employers often conflict with the interests of employees.
B. No legally accepted methods exist for calculating the risk of a job applicant's having a heart attack as a result of
being employed in any particular occupation.
C. Some jobs might involve health risks other than the risk of heart attack.
D. Employees who have a 90 percent chance of suffering a heart attack may be unaware that their risk is so great.
E. The number of people applying for jobs at a company might decline if the company, by screening applicants for
risk of heart attack, seemed to suggest that the job entailed high risk of heart attack.

59. Bank depositors in the United States are all financially protected against bank failure because the government
insures all individuals' bank deposits. An economist argues that this insurance is partly responsible for the high
rate of bank failures, since it removes from depositors any financial incentive to find out whether the bank that
holds their money is secure against failure. If depositors were more selective, then banks would need to be
secure in order to compete for depositors' money. Which of the following, if true, most seriously
weakens the economist's argument?
A. Before the government started to insure depositors against bank failure, there was a lower rate of bank failure
than there is now.
B. When the government did not insure deposits, frequent bank failures occurred as a result of depositors' fears of
losing money in bank failures.
C. Surveys show that a significant proportion of depositors are aware that their deposits are insured by the
government.
D. There is an upper limit on the amount of an individual's deposit that the government will insure, but very few
individuals' deposits exceed this limit.
E. The security of a bank against failure depends on the percentage of its assets that are loaned out and also on
how much risk its loans involve.

59.
Fact:Bank depositors in the United States are all financially protected against bank failure because the government insures all individuals' bank
deposits
Premise: Since depositors do not lose any money of the banks fail ,they do not care about the soundness of the investment the bank makes.
Conclusion: If the government did not insure the depositors money, the depositors would take more care in understanding the bank polices, thereby
making the banks more secure.

This conclusion is clearly based on the assumption that the depositors would take more care in understanding the bank polices. We need to find an
option that indicates this may not be true.

A. Strengthens the argument.

B. CORRECT. By telling us that the depositors tend to shy away form banks all together, instead of investing in more secure banks, this option clearly
weakens the conclusion by stating that the economist's plan will backfire.
C. Irrelevant. The passage already assumes this to be true.

D. Irrelevant. By stating that very few individuals' deposits exceed this limit this options does not affect the conclusion.

E. Irrelevant. The assessment of bank security is irrelevant to the argument.

58.
A. Irrelevant. The passage already states that hiring new officers will lead to increased arrests. The passage does not indicate whether this leads to a
lowered crime rate or not.

B. This restates what we already know - hiring new officers will lead to increased arrests. Rather than weakening the conclusion this strengthens it a
bit.

C. Irrelevant. The officers' salary is completely out of scope. We are discussing hiring new officiers.

D. Since this conclusion says the cost of BOTH arrests as well as convictions cannot be borne my middletown, lowered conviction rate does not help
weaken the conclusion.

E. CORRECT. By stating that hiring new officers will have a deterrent effect on crime, this option indicates that the crime rate may go down without
resulting in convictions and arrests.
58. The proposal to hire ten new police officers in Middletown is quite foolish. There is sufficient funding to pay the
salaries of the new officers, but not the salaries of additional court and prison employees to process the
increased caseload of arrests and convictions that new officers usually generate. Which of the following, if
true, will most seriously weaken the conclusion drawn above?
A. Studies have shown that an increase in a city's police force does not necessarily reduce crime.
B. When one major city increased its police force by 19 percent last year, there were 40 percent more arrests and
13 percent more convictions.
C. If funding for the new police officers' salaries is approved, support for other city services will have to be
reduced during the next fiscal year.
D. In most United States cities, not all arrests result in convictions, and not all convictions result in prison terms.
E. Middletown's ratio of police officers to citizens has reached a level at which an increase in the number of
officers will have a deterrent effect on crime.

RR wrote:Paleontologist: About 2.8 million years ago, many species that lived near the ocean floor suffered substantial population declines. These
declines coincided with the onset of an ice age. The notion that cold killed those bottom-dwelling creatures outright is misguided, however;
temperatures near the ocean floor would have changed very little. Nevertheless, the cold probably did cause the population declines, though
indirectly. Many bottom-dwellers depended for food on plankton, small organisms that lived close to the surface and sank to the bottom when they
died. Most probably, the plankton suffered a severe population decline as a result of sharply lower temperatures at the surface,
depriving many bottom-dwellers of food.

In the paleontologist's reasoning, the two portions in boldface play which of the following roles ?

(A) The first introduces the hypothesis proposed by the paleontologist; the second is a judgement offered in spelling out that hypothesis.
(B) The first introduces the hypothesis proposed by the paleontologist; the second is a position that the paleontologist opposes.
(C) The first is an explanation challenged by the paleontologist; the second is an explanation proposed by the paleontologist
(D) The first is a judgement advanced in support of a conclusion reached by the paleontologist; the second is that conclusion
(E) The first is a generalization put forward by the paleontologist; the second presents certain exceptional cases in which that generalization does not
hold good

I am sorry if this question has already been posted on the forum, but just not able to search for it. I am not sure if the search feature is not working or if
I am not searching properly. I assume that to search, you type your search string in the text box in the top right corner (Google Custom Search).

OA - A.
Any thougts why ? How do you distinguish between judgement and explanation ?

first thing: in choice (e), you have "does not hold good" ...??
that must be a mistranscription.

--

WHENEVER you have this problem type, IT IS ABSOLUTELY ESSENTIAL THAT YOU FIND THE CONCLUSION OF THE PASSAGE
FIRST.
depending on how good you are at such things, you may want to make a diagram of the passage; if you're sufficiently good at locating conclusions,
though, you may have no need to make a diagram.

the conclusion of this passage is "the cold probably did cause the population declines, though indirectly".
if you don't see why, post back and we'll explain.
once you figure out that's the conclusion, there are only two answer choices left in play: (a) and (b), the only two choices that actually say that's the
conclusion (the "hypothesis proposed by the paleontologist").
note that "a generalization" and "an explanation" are not going to represent conclusions. a "judgment" could be a conclusion, but not in the case of
choice (d), because there it's followed immediately by "...in support of X".

between (a) and (b), you don't have to think that hard. choice (a) says that the second boldface is for the conclusion, while choice (b) says the second
boldface is against the conclusion. since the former is true - the second boldface is the rationale behind the paleontologist's hypothesis - you go with
(a).

always, always find the conclusion first

Many winemakers use cork stoppers; but cork stoppers can leak, crumble, or become moldy , so that those winemakers must often discard a
significant proportion of their inventory of bottled wine. Bottlemaster plastic stoppers, which cannot leak, crumble, or mold, have long been available
to winemakers, at a price slightly higher than that of traditional cork stoppers. Cork prices, however, are expected to rise dramatically in
the near future. Clearly, therefore, winemakers who still use cork but wish to keep production costs from rising will be forced to
reconsider plastic stoppers. And since the wine-buying publics association of plastic stoppers with poor-quality wine is weakening,there is an
excellent chance that the Bottlemaster plastic stopper will gain an increased share of the marked for wine-bottle stoppers.

In the argument given, the two portions in boldface play which of the following roles?

A. The first is a judgment that has been advanced in support of a position that the argument opposes; the second is the main conclusion of the
argument.
B. The first is a judgment that has been advanced in support of a position that the argument opposes; the second is a conclusion drawn in order to
support the main conclusion of the argument.
C. The first is the main conclusion of the argument; the second provides evidence in support of that main conclusion.
D. The first is the main conclusion of the argument; the second is a restatement of that main conclusion.
E. The first is a conclusion drawn in order to support the main conclusion of the argument; the second is that main conclusion.

OA: E

can someone explain this one?

I chose A, why isn't it right?


thank u!

Need free GMAT or MBA advice from an expert? Register for Beat The GMAT now and post your question in these forums!

GMAT/MBA EXPERT

lunarpowerGMAT Instructor
Follow user
View User Profile
Joined

03 Mar 2008
Posted:

3367 messages
Followed by:

1336 members
Thanked:

2139 times
GMAT Score:

800

Mon Jan 21, 2013 2:55 am


Quote
amysky_0205 wrote:
I chose A, why isn't it right?
thank u!

choice (a) says...


The first is a judgment that has been advanced in support of a position that the argument opposes
in other words, according to choice (a), the first boldface goes AGAINST the argument.

that's not true: according to the first statement, winemakers will have to think about replacing their cork stoppers with plastic ones.
that idea provides direct support for the following part (= the claim that one particular plastic stopper will gain market share).
so, you need to pick the choice that says the first boldface is actually on the author's side. that's what choice (e) says.

_________________
Ron is a Director of Curriculum Development at Manhattan GMAT. He has been teaching various standardized tests for 20 years.

--

Pueden hacerle preguntas a Ron en castellano


Potete chiedere domande a Ron in italiano
On peut poser des questions Ron en franais
Voit esitt kysymyksi Ron:lle mys suomeksi

--

Quand on se sent bien dans un vtement, tout peut arriver. Un bon vtement, c'est un passeport pour le bonheur.

Yves Saint-Laurent

--

Learn more about ron


Thanked by: amysky_0205, rahul.sehgal@btgchampion

Free Manhattan GMAT online events - The first class of every online Manhattan GMAT course is free. Classes start every week.

tanvietGMAT Titan
Follow user
View User Profile
Joined

20 May 2008
Posted:

1404 messages
Thanked:

18 times

Sat Jan 26, 2013 1:48 am


Quote
lunarpower wrote:
amysky_0205 wrote:
I chose A, why isn't it right?
thank u!

choice (a) says...


The first is a judgment that has been advanced in support of a position that the argument opposes
in other words, according to choice (a), the first boldface goes AGAINST the argument.

that's not true: according to the first statement, winemakers will have to think about replacing their cork stoppers with plastic ones.
that idea provides direct support for the following part (= the claim that one particular plastic stopper will gain market share).
so, you need to pick the choice that says the first boldface is actually on the author's side. that's what choice (e) says.

Thank you Ron,


1.
for bf question, I normally am stucked between 2 answer choices both of which correctly describe the relations of each bf to the main conclusion but
both of which decribe differently what are bf. for example one choice describe bf1 as evidence , other choice describe bf2 as judgement. the problem
is that I see both these descritions correct and can not find the oa.

do you have experience on this problem? pls help


gmacforjyoabJust gettin' started!
Follow user
View User Profile
Joined

20 Jan 2013
Posted:

5 messages
Thanked:

1 times

Fri Sep 13, 2013 3:45 pm


Quote
lunarpower wrote:
amysky_0205 wrote:
I chose A, why isn't it right?
thank u!

choice (a) says...


The first is a judgment that has been advanced in support of a position that the argument opposes
in other words, according to choice (a), the first boldface goes AGAINST the argument.

that's not true: according to the first statement, winemakers will have to think about replacing their cork stoppers with plastic ones.
that idea provides direct support for the following part (= the claim that one particular plastic stopper will gain market share).
so, you need to pick the choice that says the first boldface is actually on the author's side. that's what choice (e) says.

Hi Ron ,

I was struck between D and E and went with D eventually . I thought the first bold part was the main conclusion . Although in option D , the second
BF description doesnt sound completely right , but kind of sounds ok , compared to option E (Unless the second BF is the main conclusion , in which
case I am wrong) . I saw multiple answers for the same question in various forums ( some of which say Ans is D and others E).
Appreciate if you can confirm on this.

Thanks !
J

GMAT/MBA EXPERT

lunarpowerGMAT Instructor
Follow user
View User Profile
Joined

03 Mar 2008
Posted:

3367 messages
Followed by:

1336 members
Thanked:

2139 times
GMAT Score:

800

Fri Sep 13, 2013 11:01 pm


Quote
gmacforjyoab wrote:
I was struck between D and E and went with D eventually . I thought the first bold part was the main conclusion .
nope.

one way to test this is to put the word "therefore" between the two statements, in both directions, and see which direction makes sense.

try it:

winemakers who still use cork but wish to keep production costs from rising will be forced to reconsider plastic stoppers
THEREFORE
there is an excellent chance that the Bottlemaster plastic stopper will gain an increased share of the marked for wine-bottle stoppers
--> this makes sense.

there is an excellent chance that the Bottlemaster plastic stopper will gain an increased share of the marked for wine-bottle stoppers
THEREFORE
winemakers who still use cork but wish to keep production costs from rising will be forced to reconsider plastic stoppers
--> this doesn't make sense.

Quote:
Although in option D , the second BF description doesnt sound completely right , but kind of sounds ok , compared to option E (Unless the second BF
is the main conclusion , in which case I am wrong) .

the second bold is the main conclusion.

Quote:
I saw multiple answers for the same question in various forums ( some of which say Ans is D and others E).

you may want to check to see whether it's actually the same problem every time. in many instances there are multiple problems that use the same
passage.

_________________
Ron is a Director of Curriculum Development at Manhattan GMAT. He has been teaching various

61. During the past year, Pro-Tect Insurance Companys total payouts on car-theft claims were
larger than the company can afford to sustain. Pro-Tect cannot reduce the number of
car-theft policies it carries, so cannot protect itself against continued large payouts that
way. Therefore, Pro-Tect has decided to offer a discount to holders of car-theft
policies whose cars have antitheft devices. Many policyholders will respond to the
discount by installing antitheft devices, since the amount of the discount will within two years
typically more than cover the cost of installation. Thus, because cars with antitheft devices are
rarely stolen, Pro-Tects plan is likely to reduce its annual payouts. In the argument above,
the two portions in boldface play which of the following roles?
A. The first rules out a certain strategy for achieving a goal; the second presents the strategy that
was adopted instead and whose effectiveness the argument assesses.
B. The first is a judgment made in support of a certain conclusion; the second is that conclusion.
C. The first has been used as a consideration to support adopting a certain strategy for achieving
a goal; the second reports a decision to adopt an alternative strategy.
D. The first provides evidence in favor of adopting a certain strategy for achieving a goal; the
second reports a decision to pursue an alternative goal.
E. The first is a consideration offered against adopting a certain strategy for achieving a goal; the
second is the main conclusion that the argument is seeking to establish

ricky question. I would opt A, though some other options also looks quite tempting.

The argument is talking about two strategies to reduce the payout:


1st, by changing the car-theft policies, which is not possible
2nd, by encouraging policyholders to install anti-theft device
Based on this the argument is giving the conclusion to opt for 2nd strategy.

Please note, this is not the main conclusion. Rather the main conclusion is in the last line "Thus, because cars with antitheft devices are rarely
stolen, Pro-Tects plan is likely to reduce its annual payouts."

A. The first rules out a certain strategy for achieving a goal; the second presents the strategy that was adopted instead and whose effectiveness the
argument assesses. -Correct
B. The first is a judgment made in support of a certain conclusion; the second is that conclusion. -First is not a judgment to support 2nd, rather
both are different strategies.
C. The first has been used as a consideration to support adopting a certain strategy
for achieving a goal; the second reports a decision to adopt an alternative strategy. -Both are different strategies, without any relation.
D. The first provides evidence in favor of adopting a certain strategy for achieving a
goal; the second reports a decision to pursue an alternative goal. -Both are different strategies, without any relation.
E. The first is a consideration offered against adopting a certain strategy for
achieving a goal; the second is the main conclusion that the argument is seeking
to establish. -2nd is a sub-conclusion, and not the main conclusion. Rather the main conclusion is in the last line.
_________________

60.
This is very similar to Q51

BF1: Some corporations shun the use of executive titles.


This explains the strategy that has been adapted by certain companies. The passage goes on to explain why this strategy has been adapted.

BF2: use of a title can facilitate an executives dealings with external businesses.

This tells us the down-side of adapting BF1 as a strategy

Main conclusion: Clearly, corporations should adopt the compromise of encouraging their executives to use their corporate titles externally but not
internally

A. The first describes a strategy that has been adopted to avoid a certain problem correct.

the second presents a drawback to that strategy correct.

B. The first describes a strategy that has been adopted to avoid a certain problem correct.

the second is a consideration raised to call into question the effectiveness of that strategy as a means of achieving that goal incorrect. We are not
discussing a goal here. The second BF is simple another fact that could cause BF to be ineffective

C. The first describes a strategy that has been adopted to avoid a certain problem correct

the second is a consideration the consultant raises in questioning the significance of that problem incorrect. the consultant does not question the
significance of the problem (the problem here is use of titles indicating position in the corporation tends to inhibit

communication up and down the corporate hierarchy

D. The first is part of an explanation that the consultant offers for a certain phenomenon incorrect. BF1 is a strategy that the consultant evaluates

E. The first describes a policy for which the consultant seeks to provide a justification incorrect. The consultant does not justify the strategy of
shunning titles but finds a middle ground.

Miscellaneous Latest Questions


95. Environmentalist: The use of snowmobiles in the vast park north of Milville creates
unacceptable levels of air pollution and should be banned.
Milville business spokesperson: Snowmobiling brings many out-of-towners to Milville in
winter months, to the great financial benefit of many local residents. So, economics dictate
that we put up with the pollution.
Environmentalist: I disagree: A great many cross-country skiers are now kept from visiting
Milville by the noise and pollution that snowmobiles generate.
Environmentalist responds to the business spokesperson by doing which of the
following?
A. Challenging an assumption that certain desirable outcome can derive from only one set of
circumstances
B. Challenging an assumption that certain desirable outcome is outweighed by negative aspects
associated with producing that outcome
C. Maintaining that the benefit that the spokesperson desires could be achieved in greater degree
by a different means
D. Claiming that the spokesperson is deliberately misrepresenting the environmentalists position
in order to be better able to attack it
E. Denying that an effect that the spokesperson presents as having benefited a certain group of
people actually benefited those people

95.

The summary of the business man's argument is due to the economic benefits, we HAVE to put up with the pollution being caused by snowmobiles.

The environmentalist says that its not just the environment that is being affected but the economy also (by stating the example of skiers)

A. CORRECT.

The environmentalist challenges that the desirable outcome (of economic benefit) can be derived only by using snowmoblies by giving an example of
where these snowmobiles actually are harming the economy.

B. The environmentalist is citing an example relevant to the economy, thereby challenging the business man's conclusion.

C. This is clearly not true. The environmentalist does not make a comparison between the economic benefits of snowmobiles versus skies.

D. This is clearly incorrect.

E. The environmentalist is not denying the economic benefits that snowboarding brings. But rather indicating that there is no need to compromise the
economy for the environment.

100.Vervet monkeys use different alarm calls to warn each other of nearby predators, depending on whether the
danger comes from land or from the air. Which one of the following, if true, contributes most to an explanation of
the behavior of vervet monkeys described above?
A. By varying the pitch of its alarm call, a vervet monkey can indicate the number of predators approaching.
B. Different land-based predators are responsible for different numbers of vervet monkey deaths.
C. No predators that pose a danger to vervet monkeys can attack both from land and from the air.
D. Vervet monkeys avoid land-based predators by climbing trees but avoid predation from the air by diving into
foliage.
E. Certain land-based predators feed only on vervet monkeys, whereas every predator that attacks vervet monkeys
from the air feeds on many different animals

100. The correct answer choice is (D). The situation in the stimulus is that
vervet monkeys use different calls depending on where predators come from.
The correct answer must explain why the calls are different (again, difference
versus similarity is an issue). Note that the stimulus does not contain a true
paradox, just an odd situation that is presented without explanation. Answer
choice (A): This answer states that vervet monkeys vary the calls in order to
indicate the number of predators, but the answer does not explain why
different calls are used for land versus air predators. This answer is attractive
because it shows that different calls can be used to indicate different things,
but it is wrong because it does not explain the behavior of the monkeys as
described in the stimulus. Answer choice (B): This answer addresses only
land-based predators and does not explain the difference described in the
stimulus. Answer choice (C): This answer states that the predators using land
attacks are different from the predators using air attacks, but this information
does not explain why vervet monkeys use different calls to indicate that fact.
Answer choice (D): This is the correct answer. Because vervet monkeys react
to predators in different ways, they would need to know if the predator was
coming by land or air. Hence, the different calls are used to tell the monkeys
whether they should climb trees or dive into the foliage. Since this answer
explains the behavior of vervet monkeys, this answer is correct. Answer
choice (E): The diet of selected predators of vervet monkeys is irrelevant and
does not help explain why vervet monkeys use different calls depending on
the direction of the attack.

STRENTHENEING IS ........REVERSE IS NOT HAPPENING


Tue Apr 26, 2011 2:24 pm

ideally this should have been written as an assumption question. this is a classic example of a
models of causation problem. researchers notice a protein correlates with a food preference,
and they assert that the protein causes the food preference. the correct answer for a problem
of this type should eliminate the possibility that the food preference caused the protein. this
is what D does here..

90. Galanin is a protein found in the brain. In an experiment, rats that consistently chose to eat fatty foods when
offered a choice between lean and fatty foods were found to have significantly higher concentrations of galanin in
their brains than did rats that consistently chose lean over fatty foods. These facts strongly support the conclusion
that galanin causes rats to crave fatty foods. Which one of the following, if true, most supports the argument?
A. The craving for fatty foods does not invariably result in a rats choosing those foods over lean foods.
B. The brains of the rats that consistently chose to eat fatty foods did not contain significantly more fat than did the
brains of rats that consistently chose lean foods.
C. The chemical components of galanin are present in both fatty foods and lean foods.
D. The rats that preferred fatty foods had the higher concentrations of galanin in their brains before they were
offered fatty foods.
E. Rats that metabolize fat less efficiently than do other rats develop high concentrations of galanin in their brains.

90. This stimulus also contains causal reasoningthe conclusion takes a


correlation and turns it into a causal relationship: G = higher concentration of
galanin in the brain, CFF = crave fatty foods, GCFF. As with all causal
arguments, once you identify the causality, you must immediately look to the
question stem and then attack. In this instance, the author simply assumes
that galanin is the cause. Why cant the fatty foods lead to higher
concentrations of galanin? Answer choice (A): If anything, this answer choice
may hurt the argument by showing that the cravings do not always lead to
choosing fatty foods. But, since the author uses the phrase consistently
chose to describe the choices of the rats, an answer stating that rats did not
invariably choose fatty foods has no effect on the argument. Answer choice
(B): This is a Shell Game answer because the test makers try to get you to
fall for an answer that addresses the wrong issue. The argument discusses
the concentration of galanin in the brains of rats; no mention is made of the
fat content of the brains of rats. This answer, which focuses on the fat content
in the brains of rats, therefore offers no support to the argument. Even though
the brain might not contain more fat, a rat could still consistently choose and
eat foods with a higher fat content. Answer choice (C): The argument is that
galanin in the brain causes rats to crave fatty foods. The fact that galanin is in
the food does not help that assertion and may actually hurt the argument.
Answer choice (D): This is the correct answer. The answer strengthens the
argument by eliminating the possibility that the stated causal relationship is
reversed: if the rats had higher concentrations of galanin prior to eating the
fatty foods, then the fatty foods cannot be the cause of the higher
concentration of galanin. As discussed earlier in the chapter, this approach
strengthens the argument by making it more likely that the author had the
original relationship correct. Answer choice (E): This answer choice hurts the
argument by suggesting that the causal relationship in the conclusion is
reversed. Remember that in Strengthen questions you can expect to see
Opposite answers, and this is one.

93. Columnist: George Orwells book 1984 has exercised much influence on a great number of this newspapers
readers. One thousand readers were surveyed and asked to name the one book that had the most influence on
their lives. The book chosen most often was the Bible; 1984 was second.

The answer to which one of the following


questions would most help in evaluating the columnists argument?

A. How many books had each person surveyed read?


B. How many people chose books other than 1984?
C. How many people read the columnists newspaper?
D. How many books by George Orwell other than 1984 were chosen?
E. How many of those surveyed had actually read the books they chose?

93. The correct answer choice is (B). The conclusion of the argument is the first
sentence: George Orwells book 1984 has exercised much influence on a
great number of this newspapers readers. The basis for this conclusion is
that 1984 was the second most named book in a survey about influential
books. The argument contains a serious error: just because 1984 came in
second in the survey does not mean that a great number of readers
selected it as influential. To illustrate this proposition, consider the following
example: Number of people surveyed = 1000, Number of people naming the
Bible as the most influential book = 999, Number of people naming 1984 as
the most influential book = 1, In this example, 1984 has come in second, but
no one would say this second place finish supports a conclusion that 1984
has exercised much influence on a great number of this newspapers
readers. You can expect the correct answer to address this issue. Answer
choice (A): The survey in the argument asks readers to name the one book
with the most influence in their lives; the number of books read does not
affect this answer. To apply the Variance Test, try opposite answers of 1
and a large number, say 10,000. These numbers will not alter the
evaluation of the argument, and thus this answer is incorrect. Answer choice
(B): This is the correct answer, but it can be difficult since the wording is a bit
unusual. The question is intended to reveal how many people selected 1984
relative to the other choices, and this addresses the issue raised in the
analysis of the stimulus. Consider how the variance test works for this answer
First try the response, 999. In this case, only one person selected
choice:
1984 as the most influential book, and the argument is greatly weakened.
Next try the response, 501. In this instance, 499 people selected 1984 as
the most influential book and the conclusion is strengthened (the other 501
people would have selected the Bible). Note that you cannot try a number
larger than 501 because that would mean that the Bible was not named most
often. Because the varied responses produce different evaluations of the
argument, this answer is correct. Answer choice (C): This answer is not
relevant to the columnists argument. Apply the Variance Test to disprove this
answer by using opposite answers of 0 and a very large number, such as 1
million. Answer choice (D): Because the argument is about Orwells 1984,
other Orwell books chosen by the readers have no impact on the argument.
Apply the Variance Test, using opposite answers of 0 and a small number
such as 10 (Orwell wrote dozens of essays, but not dozens of books).
Answer choice (E): The survey in the argument addresses influence, not the
actual reading of the book. A person might be influenced by a book like the
Bible through church teachings, etc. without actually having read the book.
To apply the Variance Test, try opposite answers of 0 and 1000.

80. On some hot days the smog in Hillview reaches unsafe levels, and on some hot days the wind blows into Hillview
from the east. Therefore, on some days when the wind blows into Hillview from the east, the smog in Hillview
reaches unsafe levels. The reasoning in the argument is flawed in that the argument
A. mistakes a condition that sometimes accompanies unsafe levels of smog for a condition that necessarily accompanies
unsafe levels of smog
B. fails to recognize that one set might have some members in common with each of two others even though those
two other sets have no members in common with each other
C. uses the key term unsafe in one sense in a premise and in another sense in the conclusion
D. contains a premise that is implausible unless the conclusion is presumed to be true
E. infers a particular causal relation from a correlation that could be explained in a variety of other ways
80. The premises of the argument contain a Formal Logic setup: HD = hot days
in Hillview, SUL = smog reaches unsafe levels, WBE = wind blows in from
the east. HDsSUL. HDsWBE. The combination of two
some statements does not yield any inferences. Yet, the author draws a
conclusion (SULsWBE ) on the basis of the relationship and you
must identify the answer that explains why this conclusion is incorrect.
that the condition of
Answer choice (A): There is no proof in the argument
WBE sometimes accompanies smog reaching unsafe levelsthat is the
mistake made by the author. The answer would be more attractive if it read
as follows: mistakes a condition (WBE) that sometimes accompanies hot
days in Hillview for a condition that sometimes accompanies unsafe levels of
smog. Answer choice (B): This is the correct answer. When two some
statements are joined, no inference can be drawn because the group
common to both may be large enough that the two sub-elements do not
overlap. For example, lets say there are 10 hot days in Hillview (HD), 1 day
when the smog reaches unsafe levels (SUL), and 1 day when the wind blows
in the east. Is it necessary that the 1 day when the smog reaches unsafe
levels is the same day that the wind blows in from the east? No, but the
argument concludes that is the case, and that error is described in this
answer choice. For reference purposes, here is the answer choice with each
abstract item identified in parentheses after the reference: fails to recognize
that one set (HD) might have some members in common with each of two
others (SUL and WBE) even though those two other sets (SUL and WBE)
have no members in common with each other. Answer choice (C): This
answer choice describes the Uncertain Use of a Term, but the argument is
consistent in its use of unsafe. Therefore, this answer is incorrect. Answer
choice (D): Each premise is plausible regardless of the truth of the
conclusion. Answer choice (E): The argument does not feature causal
reasoning. The conclusion clearly states that the two events happen together,
but there is no attempt to say that one caused the other. If you chose this
answer, try to identify the causal indicators in the argumentthere are none.

81. Astronomer: I have asserted that our solar system does not contain enough meteoroids and other cosmic debris
to have caused the extensive cratering on the far side of the moon. My opponents have repeatedly failed to
demonstrate the falsity of this thesis. Their evidence is simply inconclusive; thus they should admit that my thesis
is correct. The reasoning in the astronomers argument is flawed because this argument
A. criticizes the astronomers opponents rather than their arguments
B. infers the truth of the astronomers thesis from the mere claim that it has not been proven false
C. ignores the possibility that alternative explanations may exist for the cratering
D. presumes that the astronomers thesis should not be subject to rational discussion and criticism
E. fails to precisely define the key word meteoroids

81. This problem features an Evidence error. In this problem, the astronomer falls
into the second error from the Errors in the Use of Evidence section, where
Lack of evidence against a position is taken to prove that position is true.
Answer choice (B) describes this mistake. The astronomers argument is
structured as follows: Premise: I have asserted that our solar system does
not contain enough meteoroids and other cosmic debris to have caused the
extensive cratering on the far side of the moon. Premise: My opponents
have repeatedly failed to demonstrate the falsity of this thesis. Their evidence
is simply inconclusive. Conclusion: They [my opponents] should admit that
my thesis is correct. Answer choice (A): The argument in the stimulus does
not include a Source attack. There is a difference between stating that an
opponents argument is wrong (which is legitimate) and attacking the
character of that opponent (a Source flaw). Always look to see if the author
attacks the person or the position; a legitimate argument can sometimes
appear questionable if the author uses weighted language such as, My
opponents are deluded in believing that my thesis is incorrect. Although that
phrasing sounds like a personal attack, it is just a very strong way of stating
that the authors opponents are incorrect, and it is not a Source attack.
Answer choice (B): This is the correct answer. Answer choice (C): The
astronomers thesis asserts that meteoroids and other cosmic debris are not
the cause of the cratering on the far side of the moon. By definition, therefore,
the astronomer allows for alternate explanations of the cratering. Answer
choice (D): There is no presumption in the argument similar to the one
described in this answer. Answer choice (E): This answer describes the
Uncertain Use of a Term, but the argument does not use meteoroids in an
inconsistent way.

82. Some people believe that good health is due to luck. However, studies from many countries indicate a strong
correlation between good health and high educational levels. Thus research supports the view that good health is
largely the result of making informed lifestyle choices. The reasoning in the argument is most vulnerable to
criticism on the grounds that the argument
A. presumes, without providing justification that only highly educated people make informed lifestyle choices
B. overlooks the possibility that people who make informed lifestyle choices may nonetheless suffer from inherited
diseases
C. presumes, without providing justification, that informed lifestyle choices are available to everyone
D. overlooks the possibility that the same thing may causally contribute both to education and to good health
E. does not acknowledge that some people who fail to make informed lifestyle choices are in good health

82. The argument contains a causal conclusion that asserts that good health is
primarily caused by informed lifestyle choices (education): Premise: Some
people believe that good health is due to luck. Premise: However, studies
from many countries indicate a strong correlation between good health and
high educational levels. Conclusion: Thus research supports the view that
good health is largely the result of making informed lifestyle choices. The
author errs in assuming that the correlation mentioned in the second premise
supports a causal conclusion. Answer choice (A): A disproportionate number
of people (about one in three) select this answer. Does the argument
presume that to make an informed lifestyle choice a person must be highly
educated? The author certainly believes that high educational levels lead to
informed choices, but the answer suggests that the author thinks that the
highly educated are the only people able to make an informed choice. The
wording is too strong and this answer is incorrect. Answer choice (B): The
author specifically notes that good health is largely the result of making
informed lifestyle choices. There is no mention of poor health, nor need there
be since the argument focuses on a correlation between good health and
education. Thus, overlooking the possibility mentioned in this answer choice
is not an error. Answer choice (C): The author does not make the
presumption that informed lifestyle choices are available to everyone, just
that making good choices generally results in good health. Answer choice
This is the correct answer. Remember, the error of causality is one with
(D):

many facets, and one of those errors is assuming that no third element
caused both the stated cause and the stated effect. This answer choice
indicates that a third element (such as money) could cause both the
conditions described in the argument. Remember, if you know an error of
causality occurred in the stimulus, look for the answer that uses the words
cause or effect! This is the only answer to do so, and it is correct. Answer
choice (E): Unlike many causal conclusions, the conclusion in this argument
is not ironclad. The author specifically says that the effect is largely the result
of the cause, and that statement implicitly allows other causes to lead to the
effect, even if one does not make an informed lifestyle choice.

86. Amphibian populations are declining in numbers worldwide. Not coincidentally, the earths ozone layer has been
continuously depleted throughout the last 50 years. Atmospheric ozone blocks UV-B, a type of ultraviolet radiation
that is continuously produced by the sun, and which can damage genes. Because amphibians lack hair, hide, or
feathers to shield them, they are particularly vulnerable to UV-B radiation. In addition, their gelatinous eggs lack
the protection of leathery or hard shells. Thus, the primary cause of the declining amphibian population is the
depletion of the ozone layer.

Each of the following, if true, would strengthen the argument EXCEPT:

A. Of the various types of radiation blocked by atmospheric ozone, UV-B is the only type that can damage genes.
B. Amphibian populations are declining far more rapidly than are the populations of non-amphibian species whose
tissues and eggs have more natural protection from UV-B.
C. Atmospheric ozone has been significantly depleted above all the areas of the world in which amphibian populations
are declining.
D. The natural habitat of amphibians has not become smaller over the past century.
E. Amphibian populations have declined continuously for the last 50 years.

86. This question is much more difficult than the previous question, in part
because one of the wrong answer choices is very attractive. The conclusion
of the argument is a causal statement that the depletion of the ozone layer is
the primary cause of the declining amphibian population: DO = depletion of
the ozone layer, DA = decline of amphibian population, DODA. This
conclusion is based on the fact that the ozone layer blocks harmful UV-B
radiation, which amphibians are vulnerable to in both adult and egg form.
Although the argument mentions UV-B radiation, which may sound
impressive, the structure of the reasoning is easy to follow and no knowledge
of the radiation is needed. The conclusion is clearly stated and easy to spot
due to the indicator thus. The question stem is a StrengthenX and therefore
the four incorrect answers will each strengthen the argument. As with the
previous question, look for answers that fit the five causal strengthening
(A): This is the correct
answer types discussed earlier. Answer choice
answer. The answer fails to shed any lightpositive or negativeon the
connection between the ozone depletion and the amphibian population
decline. Because the argument is concerned with the damage done by UV-B
radiation, the fact that UV-B is the only damaging type of radiation blocked by
ozone is irrelevant. Answer choice (B): This answer choice strengthens the
when the cause is absent in non-amphibian
argument by showing that
populations, the effect does not occur (Type C). Answer choice (C): This
answer strengthens the argument by showing that the areas of ozone
depletion and amphibian decline match each other, thereby affirming the data
used to make the conclusion (Type E). Answer choice (D): This was the
answer most frequently chosen by test takers. This answer choice
strengthens the argument by eliminating an alternate cause for the effect
(Type A). Had the natural habitat become smaller over the years (from say,
human encroachment or climatic change) then that shrinkage would have
offered an alternate explanation for the decline in the amphibian population.
By eliminating the possibility of habitat shrinkage, the stated cause in the
argument is strengthened. Answer choice (E): This answer strengthens the
argument by showing that the decline of the amphibians has mirrored the
decline of the ozone layer, thereby affirming the data used to make the
conclusion (Type E).

74. Politician: Those economists who claim that consumer price increases have averaged less than 3 percent over the
last year are mistaken. They clearly have not shopped anywhere recently. Gasoline is up 10 percent over the last
year; my auto insurance, 12 percent; newspapers, 15 percent; propane, 13%; bread, 50 percent.

The reasoning
in the politicians argument is most vulnerable to criticism on the grounds that the argument
A. impugns the character of the economists rather than addressing their arguments
B. fails to show that the economists mentioned are not experts in the area of consumer prices
C. mistakenly infers that something is not true from the claim that it has not been shown to be so
D. uses evidence drawn from a small sample that may well be unrepresentative
E. attempts to persuade by making an emotional appeal

74. The correct answer choice is (D). The politicians argument is that the claims
that price increases have averaged less than 3 percent are wrong, and in
support of that position the politician cites several examples of price
increases, each of which is greater than 3 percent. As mentioned in one of
the chapter sidebars, an average is a composite number, and within the
average there can be a significant degree of variation and no single entity
need embody the exact characteristic of the average (for example, the
average weight of a 1 pound rock and a 99 pound rock is 50 pounds). In
making the argument, the politician has focused in on several individual
examples while ignoring the fact that an average is a compilation of many
different numbers. Answer choice (D) perfectly captures the essence of this
sampling error. Answer choice (A): The argument does not contain a source
or ad hominem attack. Simply stating that a position is wrong is different than
criticizing the character of that person. Answer choice (B): To claim that the
economists are wrong does not require showing that they are not pricing
experts, and hence this answer is incorrect. Answer choice (C): The politician
attempts to refute the position by providing evidence about large price
which involves facts, is different
increases for certain products. This process,
than inferring that a claim is false because it has not been shown to be true .
This answer choice would better describe an argument such as the following:
you have not proven that God exists, so there must be no God. Answer
choice (D): This is the correct answer. Citing several examples to refute an
average is a doomed strategy. Answer choice (E): There is no appeal to
emotion present; percentages are used to make the argument.

76. Cotrell is, at best, able to write magazine articles of average quality. The most compelling pieces of evidence for
this are those few of the numerous articles submitted by Cotrell that are superior, since Cotrell, who is incapable
of writing an article that is better than average, must obviously have plagiarized superior ones.

The argument is
most vulnerable to criticism on which one of the following grounds?

A. It simply ignores the existence of potential counterevidence.


B. It generalizes from atypical occurrences.
C. It presupposes what it seeks to establish.
D. It relies on the judgment of experts in a matter to which their expertise is irrelevant.
E. It infers limits on ability from a few isolated lapses in performance.

76. As always, look closely at the structure of the argumentspecifically the


relationships between the premises and conclusion. This breakdown
presents the pieces in the order given in the argument: Conclusion: Cotrell
is, at best, able to write magazine articles of average quality. Subconclusion/
Premise: The most compelling pieces of evidence for this are
those few of the numerous articles submitted by Cotrell that are superior.
Premise: Cotrell, who is incapable of writing an article that is better than
average, must obviously have plagiarized superior ones. Examine the
language in the conclusion (Cotrell is, at best, able to write magazine articles
of average quality) and the premise (Cotrell, who is incapable of writing an
article that is better than average). The two are identical in meaning, and
thus we have an argument with circular reasoning. Do not be distracted by
the plagiarism argument in the middle of the textthat is a tool used to
physically separate the conclusion and premise, making it harder to
recognize that the two are identical. Answer choice (A): The argument does
not ignore the potential counterevidence to the conclusion. The potential
counterevidence is the few articles submitted by Cotrell that are superior, and
the author dismisses them by claiming they are plagiarized. Although the
reasoning used to dismiss the good articles is flawed, it is an attempt to
address the evidence, and thus the argument cannot be said to simply
ignore the existence of potential counterevidence. Answer choice (B): This
answer choice describes an Overgeneralization. The answer is wrong
because the argument generalizes by dismissing the atypical occurrences
(the superior articles), as opposed to generalizing from them. Answer choice
(C): This is the correct answer, and one of several different ways to describe
Circular Reasoning (note that in the first problem in this set Circular
Reasoning was an incorrect answer). More often than not, when you see
Circular Reasoning it will be an incorrect answer choice, but you cannot be
complacent and simply assume it will be wrong every time you see it. This
problem proves that it does appear as the correct answer on occasion.
Answer choice (D): This answer describes an Appeal to Authority. The
answer fails the Fact Test because there is no reference to the judgment of
experts. Answer choice (E): This answer is similar to answer choice (B). The
answer starts out reasonably wellit infers limits on ability. The argument
does attempt this (depending on your definition of infer). But, does the
argument make this inference based on a few isolated lapses in
performance? No, the argument dismisses the few superior performances.
In this sense the answer is Half Right, Half Wrong. Therefore, it is incorrect.

51. Publicity campaigns for endangered species are unlikely to have much impact on the most important environmental
problems, for while the ease of attributing feelings to large mammals facilitates evoking sympathy for them, it is
more difficult to elicit sympathy for other kinds of organisms, such as the soil microorganisms on which large
ecosystems and agriculture depend.

Which one of the following is an assumption on which the argument depends?

A. The most important environmental problems involve endangered species other than large mammals.
B. Microorganisms cannot experience pain or have other feelings.
C. Publicity campaigns for the environment are the most effective when they elicit sympathy for some organism.
D. People ignore environmental problems unless they believe the problems will affect creatures with which they
sympathize.
E. An organism can be environmentally significant only if it affects large ecosystems or agriculture.

51. This is a challenging problem because two of the wrong answer choices are
attractive. The argument itself is not overly complex, but you must pay
attention to the language. Consider the conclusion of the argument: Publicity
campaigns for endangered species are unlikely to have much impact on the
most important environmental problems. Ask yourself, why is it that these
campaigns are unlikely to have much impact on the most important
problems? According to the premises, the reason is that it is more difficult to
elicit sympathy for other kinds of organisms [than large mammals]. The
reasoning shows that the author believes there is a connection between the
important problems and organisms that are not large mammals. This
Supporter connection is perfectly reflected in answer choice (A), the correct
remember to look
answer. Again, when faced with an Assumption question,

for connections between rogue elements in the argument, and then seek that
connection in the answer choices. Answer choice (B): The argument is about
eliciting sympathy, and no assumption is made about microorganisms
This is a Shell Game answer. (M SHELL GAME ANSWER
experiencing pain. Answer choice (C):
CHANGES THE SCOPE ...GIVES ANSWER ABOUT SOMETHING ELSE The
conclusion is specific about publicity
campaigns for endangered species as
they relate to environmental problems. This answer refers to publicity
campaigns in generala different concept. It may be that the most effective
publicity campaign for the environment has nothing to do with organisms.
Consequently, this answer is not an assumption of the argument. Answer
choice (D): This answer choice is worded too strongly and is an Exaggerated
answer. Ignore goes further than what the author implies. The author
indicates that it is more difficult to elicit sympathy for other kinds of
organisms, but the author does not say it is impossible to get sympathy from
individuals if a non-large mammal is involved. Further, the argument is
specific about the impact on the most important problems, and this answer
goes well beyond that domain. Answer choice (E): The microorganisms
discussed at the end of the argument are an example (such as); therefore,
the author does not assume this type of relationship must be true in order for
the conclusion to be true.

57. Speaker: Contemporary business firms need to recognize that avoiding social responsibility leads to the gradual
erosion of power. This is Davis and Blomstroms Iron Law of Responsibility: In the long run, those who do not use
power in a manner which society considers responsible will tend to lose it. The laws application to human
institutions certainly stands confirmed by history. Though the long run may require decades or even centuries in
some instances, society ultimately acts to reduce power when society thinks it is not being used responsibly.
Therefore, a business that wishes to retain its power as long as it can, must act responsibly.

Which one of the


following statements, if true, most weakens the speakers argument?
A. Government institutions are as subject to the Iron Law of Responsibility as business institutions.
B. Public relations programs can cause society to consider an institution socially responsible even when it is not.
C. The power of some institutions erodes more slowly than the power of others, whether they are socially responsible
or not.
D. Since no institution is eternal, every business will eventually fail.
E. Some businesses that have used power in socially responsible ways have lost it.

57. This problem is similar in form to the carpet market problem. The conclusion
appears at the end and is conditional in nature: a business that wishes to
retain its power as long as it can, must act responsibly. This relationship can
be diagrammed as: WRP = business wishes to retain power as long as
possible, AR = act responsibly. WRPAR. Hopefully, you identified this
conclusion as conditional when you read the stimulus. As you read the
question stem, you should have immediately pre-phrased an answer that
would allow the sufficient condition to occur without the necessary condition,
namely that a business that wishes to retain power does not necessarily have
to act responsibly. Let us examine the answer choices with this idea in mind:
Answer choice (A): Because this answer addresses government institutions,
this cannot hurt the conclusion, which is about businesses. If anything, this
may slightly support the argument. In the middle of the stimulus, the Speaker
mentions that The laws application to human institutions certainly stands
confirmed by history. This answer affirms that statement by adding
governments to the named list of human institutions. Answer choice (B): This
is the correct answer. If
a public relations program can cause society to think
an institution is socially responsible even when it is not, then an institution
that wishes to retain power could act irresponsibly and then get a public
relations firm to cover up the activities. In this way, the institution could wish
to retain power but not act responsibly. Since this scenario allows the
sufficient condition to occur without the necessary, this weakens the
argument. Answer choice (C): Many students hold this answer choice as a
Contender. The answer is incorrect because the stimulus contemplates
varying rates of power retention, especially between socially responsible and
non-socially responsible institutions. If you read this answer thinking that the
stimulus indicated socially responsible institutions do not lose power if
socially responsible, then you made a quasi-Mistaken Reversal of the
stimulus. There is never a presumption in the argument that power can be
held indefinitely. If there were, this answer would be much more attractive.
Answer choice (D): The conclusion is clear in saying, a business that wishes
to retain power as long as it can... The italicized phrase allows for the idea
that businesses will eventually lose power and ultimately fail. Thus, this
answer does not hurt the argument. Answer choice (E): This is another
attractive answer, and one that lured in many test takers. The answer states
that even though some businesses acted responsibly (AR), they did not
retain power (RP). If this difference between retaining power and wishing to
retain power (WRP) is ignored, then this answer can be seen as attacking the
Mistaken Reversal of the conclusion. As you learned from the discussion of
answer choice (C) of the carpet market question, attacking the Mistaken
Reversal of the conclusion does not hurt the conclusion. However, this
answer is attractive because not only does it address elements of the
conclusion, it also appears as the final answer choice. A test taker who did
not like any of the earlier answers would find this answer quite attractive.

60. Loggerhead turtles live and breed in distinct groups, of which some are in the Pacific Ocean and some are in the
Atlantic. New evidence suggests that juvenile Pacific loggerheads that feed near the Baja peninsula hatch in
Japanese waters 10,000 kilometers away. Ninety-five percent of the DNA samples taken from the Baja turtles
match those taken from turtles at the Japanese nesting sites.

Which one of the following, if true, most seriously


weakens the reasoning above?
A. Nesting sites of loggerhead turtles have been found off the Pacific coast of North America several thousand
kilometers north of the Baja peninsula.
B. The distance between nesting sites and feeding sites of Atlantic loggerhead turtles is less than 5,000 kilometers.
C. Loggerhead hatchlings in Japanese waters have been declining in number for the last decade while the number of
nesting sites near the Baja peninsula has remained constant.
D. Ninety-five percent of the DNA samples taken from the Baja turtles match those taken from Atlantic
loggerhead
turtles.
E. Commercial aquariums have been successfully breeding Atlantic loggerheads with Pacific loggerheads for the last
five years

60. The argument uses the premise that Baja turtles and Japanese turtles share
ninety-five percent of their DNA to conclude that Baja turtles hatch in
Japanese waters 10,000 kilometers away. This sounds like convincing
statistical evidence unless you realize that many organisms share DNA. For
example, humans and chimpanzees share about 98% of their DNA (we share
about 75% of our DNA with dogs, for that matter). Since Baja and Japanese
turtles come from the same species, it is not surprising that they would share
a high percentage of their DNA. Regardless of whether or not you saw this
connection, you should have been skeptical of the reference to juvenile
turtles traveling 10,000 kilometers. Such a lengthy trip by a juvenile animal is
unlikely, and calls into question the soundness of the argument. Answer
choice (A): This answer does not impact the argument because no details
DNA or otherwiseare given about the turtles at these nesting sites off the
Pacific coast of North America. Answer choice (B): The fact that Atlantic
turtles have nesting and feeding sites no more than 5,000 kilometers apart
does not attack the argument because the argument is about Baja turtles.
Answer choice (C): This answer attempts to weaken the argument by
inducing you to conclude that if the Japanese hatchlings are declining but
Baja sites are constant, then the Baja sites cannot be supplied by the
Japanese hatchlings. But, the answer choice moves from the number of
hatchlings to the number of sites. Even with a declining number of hatchlings,
the number of sites could remain constant, albeit with fewer turtles at each.
Because of this possibility, the answer does not undermine the argument.

This is the correct answer. The answer shows that all


Answer choice (D):

turtles in the argument have the same ninety-five percent DNA,


meaning that
the Baja turtles did not have to take the 10,000 kilometer trip. Answer choice
(E): The breeding between species was not an issue in the stimulus.

ALWAYS ....PREMISE + CONCLUSION+ SOME REASINING TO SUPPORT ( SPECIFIC PREMISE)

NOW THIS SPECIFIC PREMISE IS ALWAYS TRU......AND AND CONCLUSION NEED TO BE


CONTRADCITED ............IN THIS MANNER JSUT COUNTER THE THE CONCLSUION AND
REDUCE THE COUNTERS BY KEEPING THE SPECIFIC PREMISE ALIVE ..........

64. Lobsters and other crustaceans eaten by humans are more likely to contract gill diseases
when sewage contaminates
their water. Under a recent proposal, millions of gallons of local sewage each day would be
rerouted many
kilometers offshore. Although this would substantially reduce the amount of sewage in the
harbor where lobsters
are caught, the proposal is pointless, because hardly any lobsters live long enough to be
harmed by those diseases.

Which one of the following, if true, most seriously weakens the argument?
A. Contaminants in the harbor other than sewage are equally harmful to lobsters.
B. Lobsters, like other crustaceans, live longer in the open ocean than in industrial harbors.
C. Lobsters breed as readily in sewage- contaminated water as in unpolluted water.
D. Gill diseases cannot be detected by examining the surface of the lobster.
E. Humans often become ill as a result of eating lobsters with gill diseases

----Notice the scope shift, when he says "hardly any lobsters live long enough to be harmed by those
diseases. " as evidence for the "pointless" conclusion.

We are talking about Lobsters contracting disease. Lobsters may not be harmed by those diseases but
they can still be carriers. Hence E is correct.

64. This is a great separator question, and approximately one in three students
answers this question correctly. However, some students are able to
annihilate this question because they see a reference in the first line that
That reference is to lobsters
raises an important issue that goes unanswered.
eaten by humans. The argument asserts that diverting the sewage in the
harbor is a moot point because hardly any lobsters live long enough to be
harmed by the diseases caused by the sewage. This may be, but what about
the humans who eat the lobsters that live in the sewage-contaminated
The conclusion of the
environment? The author fails to address this point.
argument is near the end: the proposal is pointless, and this is based on the
premise that hardly any lobsters live long enough to be harmed by those
diseases. Answer choice (A): The argument is based on the sewage
contamination of the harbor. Although other contaminants may be present,
they are not addressed by the argument, and thus this answer does not
undermine the authors position. Answer choice (B): This answer has no
impact because the argument is about lobsters that are caught in the harbor.
So, while lobsters in the open ocean may live longer, the authors point about
lobsters in the harbor not living long enough to contract a gill disease is
untouched. Answer choice (C): The issue is not breeding frequency but
longevity. So, while we are pleased to hear that lobsters in sewagecontaminated
waters breed frequently, this fact does not impact an argument
based on the age and disease contraction. Answer choice (D): Although
whether the lobsters contract a gill disease is a critical issue in the argument,
the method of determining whether a lobster has a disease is not a critical
issue. Again, keep in mind the heart of the argument: Premise: hardly any
lobsters live long enough to be harmed by those diseases. Conclusion: the
proposal [to reroute harbor sewage] is pointless. Nothing in that argument
concerns the detection of the gill diseases. Answer choice (E): This is the
correct answer. As discussed above, the author fails to address the effect of
the contaminated lobsters on humans who consume them, and this answer
attacks that hole. If humans become ill as a result of eating lobsters with gill
diseases, and gill diseases are more likely to arise when the lobsters live in
the sewage-contaminated waters, then the conclusion that the proposal is
pointless is incorrect.

65. People with high blood pressure are generally more nervous and anxious than people who do not have high blood
pressure. This fact show that this particular combination of personality traitsthe so-called hypertensive personality
is likely to cause a person with these traits to develop high blood pressure.

The reasoning in the argument is most


vulnerable to criticism on the grounds that the argument

A. fails to define the term hypertensive personality


B. presupposes that people have permanent personality traits
C. simply restates the claim that there is a hypertensive personality without providing evidence to support that
claim.
D. takes a correlation between personality traits and high blood pressure as proof that the traits cause high blood
pressure.
E. focuses on nervousness and anxiety only, ignoring other personality traits that people with high blood pressure
might have

65. Premise: People with high blood pressure are generally more nervous and
anxious than people who do not have high blood pressure. Premise: This
particular combination of personality traits is called the hypertensive
personality. Conclusion: The hypertensive personality is likely to cause a
person to develop high blood pressure. The premises indicate that certain
individuals have both high blood pressure and the hypertensive personality.
From this information we cannot draw any conclusions, but the author makes
the classic GMAT error of concluding that one of the conditions causes the
other. Your job is to find the answer that describes this error of reasoning.
From the Situations That Can Lead to Errors of Causality discussion, the
scenario in this stimulus falls under item 2Two (or more) events occur at
the same time. As described in that section, While one event could have
caused the other, the two events could be the result of a third event, or the
two events could simply be correlated but one does not cause the other.
Thus, you should search either for an answer that states that the author
forgot that a third event could have caused the two events or that the author
mistook correlation for causation. Answer choice (D) describes the latter.
Answer choice (A): This is an Opposite answer because the stimulus defines
the hypertensive personality as one with the traits of nervousness and
anxiety. Answer choice (B): The permanence of the traits is not an issue in
the stimulus. Answer choice (C): Although the argument does act as
described in this answer choice, this is not an error. On the GMAT, authors
have the right to make premises that contain certain claims. Remember, the
focus is not on the premises but where the author goes with the argument
once a premise is created.
Answer choice (D): This is the correct answer.
The conclusion can be diagrammed as: HP = hypertensive personality, HBP
= high-blood pressure, HPHBP. This answer choice describes a classic
error of causality: two events occurring simultaneously are mistakenly
interpreted to be in a causal relationship. There are many other possibilities
for the arrangement: the two events could be caused by a third event (for
example, genetics could cause both a hypertensive personality and high
blood pressure), the events could be reversed (the high blood pressure could
actually cause the hypertensive personality), or there may be situations
where the two do not occur together. Answer choice (E): Although the
argument does act as described in this answer choice, this is not an error.
The author is allowed to focus on nervousness and anxiety to the exclusion
of other traits. To analogize, imagine a speaker says, The Kansas City
Royals have bad pitching and this makes them a bad team. The Kansas City
Royals might also wear blue, but the speaker is not obligated to mention that
trait when discussing why the Royals are a bad baseball team. In much the
same way, the author of this stimulus is not obligated to mention other traits
people with high blood-pressure may have.

66. High school students who feel that they are not succeeding in high school often drop out before graduating and go
to work. Last year, however, the citys high school dropout rate was significantly lower than the previous years
rate. This is encouraging evidence that the program instituted two years ago to improve the morale of high school
students has begun to take effect to reduce dropouts.

Which one of the following, if true about the last year, most
seriously weakens the argument?

A. There was a recession that caused a high level of unemployment in the city.
B. The morale of students who dropped out of high school had been low even before they reached high school.
C. As in the preceding year, more high school students remained in school than dropped out.
D. High schools in the city established placement offices to assist their graduates in obtaining employment.
E. The anti-dropout program was primarily aimed at improving students morale in those high schools with the
highest dropout rates

66. The argument concludes that a program instituted two years ago to increase
morale has ultimately caused the recent decrease in high school dropouts.
You must always recognize a causal conclusion when one is presented to
you! Whenever you encounter a causal conclusion, ask yourself if the
relationship must be as stated by the author or if another explanation can be
found. In simplified form, the conclusion appears as follows: P = program to
raise high school morale, RD = reduction in dropouts, PRD. Regardless of
the question asked, this assessment is helpful. The question stem asks you
to weaken the argument, and according to the How to Attack a Causal
Conclusion section there are five main avenues of attack you should be
prepared to encounter. The correct answer, (A), falls into one of the most
frequently occurring of those categories: the alternate cause. Answer choice
(A): This is the correct answer. The answer attacks the conclusion by
introducing an alternate cause: it was not the morale program that led to a
decrease in high dropouts, but rather the fact that no jobs were available for
individuals contemplating dropping out of high school. The job availability
factor is important because the first sentence of the stimulus indicates that
high school students who drop out go to work. Thus, if a recession led to a
high level of unemployment, this could cause high school students to rethink
dropping out and stay in school. Answer choice (B): At best, this answer
confirms that some of the high school students had a low morale, and in that
sense, the answer strengthens the argument. At worst, the answer choice is
irrelevant. Answer choice (C): The argument indicates that the dropout rate is
lower relative to the preceding year; there is no claim that the dropout rate
ever exceeded the retention rate. Thus, to suggest that more students stayed
in school than dropped out has no effect on the argument. Answer choice
(D): This is a Shell Game answer. The stimulus refers to high school
dropouts. This answer choice refers to high school graduates. Answer choice
(E): The argument uses information about the citys overall dropout rate.
Therefore, the target high schools of the anti-dropout program are irrelevant.

69. Medical researcher: As expected, records covering the last four years of ten major hospitals indicate that babies
born prematurely were more likely to have low birth weights and to suffer from health problems than were babies
not born prematurely. These records also indicate that mothers who had received adequate prenatal care were
less likely to have low birth weight babies than were mothers who had received inadequate prenatal care. Adequate
prenatal care, therefore, significantly decreases the risk of low birth weight babies.

just plot it and u will see

premature ..............................low wight


low weight ..............bad prenatal care

conlcuion is low weight ...pre natal care ..................................if prematurity is


false laso it does not effect
Which one of the following, if
true, most weakens the medical researchers argument?

A. The hospital records indicate that many babies that are born with normal birth weights are born to mothers who
had inadequate prenatal care.
B. Mothers giving birth prematurely are routinely classified by hospitals as having received inadequate prenatal care
when the record of that care is not available.............it shows a statitical problem
C. The hospital records indicate that low birth weight babies were routinely classified as having been born prematurely.....does not
show a problem .
D. Some babies not born prematurely, whose mothers received adequate prenatal care, have low birth weights.
E. Women who receive adequate prenatal care are less likely to give birth prematurely than are women who do not
receive adequate prenatal care.

69. The correct answer choice is (B). The premises contain correlations, and the
conclusion makes a causal claim: PC = adequate prenatal care, DR =
decrease risk of low birth weight babies. PCDR. The question stem asks
you to weaken the argument, and the correct answer falls into one of the five
basic methods for weakening a causal argument. Answer choice (A): The
conclusion specifically states that mothers who had received adequate
prenatal care were less likely to have low birth weight babies than mothers
who had received inadequate prenatal care. Thus, although mothers who
received inadequate prenatal care have a higher likelihood of having low birth
weight babies, this likelihood still allows for many babies to be born of normal
weight. In a later chapter we will explore the ways the GMAT uses numbers
and statistics to confuse test takers, but for now, consider this analogy: The
Detroit Tigers are more likely to lose a baseball game than any other team,
but even so, they can still win a number of games. In the same way, the
aforementioned mothers may be more likely to have low birth weight babies,
but they can still give birth to babies of normal weight. Hence, answer choice

(B): This is the correct


(A) does not attack the argument. Answer choice

answer. The answer choice falls into the category of Showing a


statistical
problem exists with the data used to make the causal statement. By
indicating that all mothers without prenatal care records are
automatically
classified as mothers receiving inadequate prenatal care, the answer
undermines the relationship in the argument because the data used to
make
the conclusion is unreliable. Answer choice (C): The conclusion is about low
birth weight babies, not premature babies. Even if low birth weight babies
were routinely classified as premature, that would not affect the conclusion.
Answer choice (D): Similar to answer choice (A), the likelihoods discussed in
the stimulus allow for this possibility. Hence, this answer cannot hurt the
argument. Answer choice (E): If anything, this answer strengthens the
argument since it shows that adequate prenatal care has a powerful positive
effect.

A certain baseball team has just completed its season. In stadiums that seat 20,000 or fewer people, the team
averaged 1 home run per game; in stadiums that seat between 20,000 and 40,000 people, the team averaged 2
home runs per game; and, in stadiums that seat 40,000 or more people, the team averaged 3 home runs per
game. Obviously, the excitement of playing in front of large crowds motivated the team to hit more home runs.
Assuming that all stadiums during the season were filled to capacity, which of the following, if true, most undermines
the argument above?
The teams leading home run hitter hit more home runs in mid-sized stadiums than in large stadiums.
The fans in the larger stadiums often cheered against the team.
The team averaged only 2 home runs per game when playing in the leagues largest stadium.
In order to create seating for the additional fans, the outfield walls in the larger stadiums were constructed closer
to home base.
The teams announcer cited crowd noise as a major motivator for the team.

1.
On average, the team hit more home runs playing in front of larger crowds than
in front of smaller crowds. The argument attributes this statistic to the motivation
that comes from playing in front of larger crowds. In order to undermine this
conclusion, look for another reason to explain why more home runs were hit in
front of larger crowds.
(A) The argument makes a claim about the collective behavior of the team. This
collective claim does not preclude certain individuals from hitting fewer home
runs in larger stadiums.
(B) The claim made in the argument is based on the size of the crowd in each
stadium. For whom the fans cheered is irrelevant to the argument.
(C) Similar to answer choice A, this choice cites one specific example of
contradictory information, while the argument is based on the average behavior
of the team throughout the entire season. The does not strongly undermine that,
on average, the team was motivated by larger crowds.
(D) CORRECT. This choice explains that the larger stadiums actually have
different dimensions from the smaller stadiums. In order to accommodate a
larger number of fans, the outfield walls are closer to the batters. Thus, it is very
possible that the greater number of home runs is due to the fact that the ball
does not have to travel as far in larger stadiums.
(E) The announcers opinion is not relevant to the argument, and, even if it were,
this choice would strengthen the argument.

5. Political Analyst: Because our city is a border city, illegal immigration is an important issue in the current race for
mayor. Of the two candidates for mayor, one supports a plan that would attempt to deport the citys 9,000 illegal
immigrants and the other does not. Surveys consistently show that about 60% of the citys residents are opposed
to the plan, while about 35% are in support of the plan. Therefore, the candidate who does not support the plan
will win the election for mayor.

All of the following statements weaken the analysts argument, EXCEPT:

In the city at issue, most voters make their voting decisions based on the candidates positions on abortion.
Of the 35% of residents who support the plan, some are willing to consider alternate plans for addressing illegal
immigration.
Many of the residents who oppose the plan are not registered voters.
The candidate who supports the plan is the incumbent mayor, and has been elected to four consecutive terms
despite taking controversial positions on many important issues.
Just under 30% of the citys residents are illegal immigrants who cannot vote

5.
The analyst argues that the mayoral candidate who opposes the deportation plan
will win the governors race because 60% of city residents also oppose the plan.
The analyst assumes that a majority of residents will vote for this candidate
based on his position on illegal immigration. Any statement that calls this
assumption into question will weaken the argument. We are looking for the one
statement that does NOT call this assumption into question.
(A) This statement calls into question the assumption that voters will cast their
ballots based on the illegal immigration issue. Therefore, this statement
weakens the analyst's argument.
(B) CORRECT. This does not weaken the argument. In fact, if some of those
who support the plan are willing to reconsider, they may ultimately oppose the
original plan and decide to vote for the candidate who is also in opposition. If
anything, this would help justify the analyst's claim that the candidate who
opposes the plan will win the election.
(C) This statement calls into question the assumption that a majority of residents
will vote for the candidate who opposes the plan. If many of these residents are
not registered voters, they will not be able to vote, regardless of their position on
the immigration issue. This weakens the argument.
(D) This calls into question the assumption that the residents will vote based on
the illegal immigration issue. This statement shows that voters have a history of
voting for the incumbent despite his controversial position on important issues. It
is possible that the voters will again vote for the incumbent, even if he has taken
an unpopular position on the illegal immigration issue. This weakens the
argument.
(E) If just under 30% of the residents are illegal immigrants, it is likely that many
of the 60% in opposition to the plan are actually illegal immigrants themselves. If
these people cant vote, it is less likely that the candidate who opposes the plan
will win.

3. Studies in restaurants show that the tips left by customers who pay their bill tend to be larger when the bill is
presented with the servers name hand-written on the bill. Psychologists hypothesize that simply seeing a handwritten
name makes many consumers feel more of a personal identification with the server, encouraging larger
tips.

Which of the following, if true, most strongly supports the psychologists interpretation of the studies?
GIVING ONE MORE EVIDENCE OF SIMILAR TYPE......

The effect noted in the studies applies to patrons paying with either credit cards or cash.
Nametags for servers have not been shown to have any effect on the size of the bill.........there is a scope change ....size of
the BILL....do not stretch it to the size of the tip inturn ,,,,
Greeting card companies have found that charities which send holiday cards with handwritten signatures are more
likely to receive donations than those which send cards with printed signatures.
The studies indicated much larger average tips if the customer ordered alcoholic beverages with his or her meal.
Many of the restaurants in which the studies were conducted are located in tourist areas, where people are
traveling for leisure activities

3.
This argument concerns a potential explanation for larger tips on the part of
restaurant patrons. The explanation provided is that customers are more
generous toward servers that leave their hand-written name on the bill due to a
greater degree of personal identification with the server, which encourages larger
tips.

The correct answer will either support the fact that a hand-written name
strengthens personal identification, or that personal identification encourages
larger tips.

(A) The fact that the effect applies equally regardless of the method of payment
is not relevant to the conclusion.
(B) The argument does not address the size of the bill; rather, it addresses the
size of the tip. This choice is irrelevant.
(C) CORRECT. This answer choice provides further evidence that a handwritten
name or signature generates a greater form of personalization and
emotional connection among recipients, leading to more donations.
(D) The impact of alcoholic beverages on tipping behavior is irrelevant.
(E) The location of the restaurants and leisure pursuits of the patrons are
irrelevant to the argument.

5. More and more companies have begun to consume less energy by making themselves more efficient. Over time,
these efforts could place the United States at the forefront of an emerging global market for cleaner technologies.
Such efforts are also essential to tackling the two big energy-related issues of the age: global warming and the
dependence on precarious supplies of oil. The federal government should encourage these efforts by providing the
necessary incentives, whether as loans, direct grants or targeted tax breaks.

ASK WHY ...WHY SUCH A CONCLUSION...AND U WILL GET THE NASWER WHICH
WILL BE REFLECTED IN THE OPTIONS
WHY TAX BREAKS DIRECTS GRANST ETC .....WHY ...BECAUSE THEY
WILL SUPPORT THE ENERGY RELATED EFFORTS
Which of the following, if true,
provides the most effective support for the argument?

On the average, Canadian companies are more energy efficient than those in the United States.
Experts believe that energy efficiency could lower the energy use of the United States to the level of 1995.
In the past, government incentives have made advances in energy conservation feasible, especially in the auto
industry.
The dependence on foreign oil is a greater problem in the present than global warming.
The market for cleaner technologies is currently relatively small because of the infrastructure requirements

5.
The argument claims that federal incentives should be provided to encourage
energy efficiency. The argument also notes that companies are already working
in this direction and that this trend will ease the environmental and energy
pressures that currently trouble the world. Supporting this argument could involve
providing evidence of possible success for these efforts toward their goals.
(A) This choice is an irrelevant comparison. That Canadian companies are more
efficient has no bearing on efforts in the United States or the role of government
incentives.
(B) This choice does not strengthen the claim. Experts' claims are not the same
as reality. Furthermore, the choice does not say whether reducing energy use to
the 1995 level is a significant decrease, nor does it provide any information to
strengthen the link between government incentives and reduced energy use.
(C) CORRECT. This choice provides evidence that government incentives are
effective. Thus, this choice confirms an assumption that the conclusion is
feasible.
(D) This choice is an irrelevant distinction. It does not matter to the arguments
conclusion if one of these issues is a greater problem than the other in the
present.
(E) The passage asserts that the United States should be at the forefront of an
emerging market for cleaner technologies; i.e., the market will be significant in
the future. The size of the market at present is irrelevant to the argument.

WORD VERIFICATION ISN STRENGTHEN AND THOUGH VERIFICATION AND ASSUMPTION CHECK IN WEAKNER

9. Officials of the Youth Hockey League and parents of players in the league have become concerned with the number
of flagrant fouls occurring during league games. This past season, the number of flagrant fouls was double the
number from the season before. League officials plan to reduce the number of such fouls during the coming
season by implementing mandatory suspensions for players who commit flagrant fouls.

Which of the following


statements, if true, provides the best evidence ( NEW INFO WILL BE GIVE ) that the officials plan will be effective?

Most serious injuries occurring during league games are a direct result of flagrant fouls.
League referees have been trained to recognize flagrant fouls and to report incidents involving such fouls.
Parents of players in the league are in support of mandatory suspensions for flagrant fouls.
A similar league suspends players for committing flagrant fouls; this league has a relatively low incidence of
flagrant fouls when compared with the Youth Hockey League.
Most players in the league strive to be selected for the All-Star team, and league rules state that no player with a
record of suspension shall be selected for the All-Star team.

(E) The number of students attending charter vs. non-charter schools has no
bearing on the conclusion. The conclusion focused on student performance on
average, thereby eliminating raw numbers of students as relevant to this
measure.
9.
League officials plan to reduce the number of flagrant fouls by implementing
mandatory suspensions for players who commit such fouls. This plan will work
only if the punishment serves to deter players from committing flagrant fouls.
(A) The cause of injuries has no bearing on whether suspensions will deter
players from committing flagrant fouls.
(B) While the referees effectiveness in recognizing and reporting flagrant fouls
will surely aid in the implementation of the new policy, this has no bearing on
whether the policy will deter players from committing flagrant fouls.
(C) The parents opinion has no bearing on whether the suspensions will deter
players from committing flagrant fouls.
(D) While we might conclude that the other, similar league has a low incidence of
flagrant fouls because it suspends players who commit such fouls, we have no
evidence to show that the suspensions actually deter players from committing
fouls. It is entirely possible that the other league has a low incidence of flagrant
fouls for other reasons. For example, maybe the players in the other league are
just inherently less aggressive.

(E) The number of students attending charter vs. non-charter schools has no
bearing on the conclusion. The conclusion focused on student performance on
average, thereby eliminating raw numbers of students as relevant to this
measure.
9.
League officials plan to reduce the number of flagrant fouls by implementing
mandatory suspensions for players who commit such fouls. This plan will work
only if the punishment serves to deter players from committing flagrant fouls.
(A) The cause of injuries has no bearing on whether suspensions will deter
players from committing flagrant fouls.
(B) While the referees effectiveness in recognizing and reporting flagrant fouls
will surely aid in the implementation of the new policy, this has no bearing on
whether the policy will deter players from committing flagrant fouls.
(C) The parents opinion has no bearing on whether the suspensions will deter
players from committing flagrant fouls.
(D) While we might conclude that the other, similar league has a low incidence of
flagrant fouls because it suspends players who commit such fouls, we have no
evidence to show that the suspensions actually deter players from committing
fouls. It is entirely possible that the other league has a low incidence of flagrant
fouls for other reasons. For example, maybe the players in the other league are
just inherently less aggressive.

7. In the 18th and 19th centuries, it was believed in many coastal American cities that the waterfront was an
undesirable location for residential buildings. As a result, much of the waterfront in these cities was never developed
aesthetically and instead was left to industry and commerce. Today, however, waterfront properties are generally
seen as prestigious, as evidenced by the large sums paid for homes along the beach front. A developer who
wishes to make a large profit would be wise to buy urban waterfront lots and erect residential buildings on them.

AGAIN ASK WHY DEVELOPER INTERESTED IN PROFITS WOULD BE WISE TO BUY URBAN WATERFRONT
AND ERECT BUILDINGS ON THEM

BECASUE PEOPLE WILL BE ABLE TO PAY THEM THE REQUISITE AMOUNT

Which of the following, if true, most strongly supports the claim made about urban waterfront properties?

People today have more money, relatively speaking, to spend on real estate than they did in previous centuries.
Homeowners will be willing to spend large sums on residential properties in traditionally industrial
or commercial
districts.
Many urban waterfront lots are available for purchase.
Many coastal American cities are encouraging developers to rehabilitate the waterfront through tax incentives.
Properties in interior residential districts in coastal American cities are significantly more expensive than those
along the waterfront

7.
The conclusion is that a developer who wishes to make a large profit would be
wise to buy urban waterfront lots and erect residential buildings on them. The
basis for that claim is that people pay large sums for beach front homes. We are
asked to strengthen this argument.
(A) This choice states that people have more buying power today than in
previous centuries. This does not strengthen the claim that a developer will make
money on urban waterfront properties.
(B) CORRECT. This choice states that homeowners will be willing to spend large
sums of money on residential properties in traditionally industrial or commercial
districts. Since we know from the argument that urban waterfronts have
traditionally been industrial, this fact strengthens the claim that a developer can
make a profit on urban waterfront properties.
(C) This choice states that many urban waterfront lots are available for purchase.
This does not suggest, however, that a developer will be able to sell them after
he or she builds on them.
(D) This choice states that many coastal cities are giving tax breaks to
developers who rehabilitate the waterfront. But this does not suggest that anyone
will buy the developed properties.
(E) This choice states that properties in the interior of cities are more expensive
than those on the waterfront. Although waterfront properties are therefore
cheaper to acquire, this does not necessarily mean that a developer can make a
profit after buying such properties.

TRY N REREAED TO MATCH SIMILAR INFO ....IN THE STRENGTHEN QUESTION

11. Market Analyst: Recent research confirms that the main cause of bad breath is bacteria build-up on the tongue.
The research also concludes that tongue scrapers, when used properly, can eliminate up to 40% of the bacteria
from the tongue. As the effectiveness of tongue scrapers becomes more widely known, the market for less
effective breath freshening products, such as mints, gums, and sprays, will decline significantly. Which of the
following provides the best evidence that the analysts argument is flawed....... MEANS WEAKENE IT ...........SO
BASICALLY MAKE THE CONCLUSION OPPSOSITE AND THEN ASK WHY ,,,,,,..........GUMS
SPARYS ETC WILL NOT DEVCLINE WHY ........AS THE EFFETIVEMENSS BECOMES MORE WIDLY
KNOWN ...................BECAUSE IT IS ALREADY WIDELY KNOWN .........FINISH ?..............WHEN
OPPSOING THE CONCLSUION ANDREADING A AS IT IS ........JUST TAKE THE BASIS ALONG
WITH IT ............AND THEN ASK WHY ....THAT WOULD HELP ELIMINATE SEVERAL OTHER
CHOICES OF THE ANSWER
Some breath freshening products are advertised to eliminate up to 30% of the bacteria from the tongue.
Tongue scrapers have already been on the market for a number of years.
Many dentists recommend regular flossing, and not the use of the tongue scraper, to combat bad breath.
A recent survey shows that 94% of those who regularly purchase breath freshening products are aware of the
effectiveness of the tongue scraper.
Some people buy breath freshening products for reasons other than to fight bad breath.

11.
The market analyst concludes that the market for breath freshening products will
decline as the effectiveness of the tongue scraper becomes more widely known.
To show that this argument is flawed, we must attack one of two assumptions:
that consumers are primarily interested in products that are most effective in
fighting bad breath, or that consumers are currently unaware of the effectiveness
of the tongue scraper.
(A) This statement does not attack either one of the assumptions. In fact, it may
actually strengthen the argument by pointing out that some breath freshening
products are less effective than tongue scrapers at eliminating bacteria.
(B) This statement could weaken the argument by attacking the second
assumption (consumers are currently unaware of the tongue scraper). If the
tongue scraper has been on the market for a while, maybe consumers are
already aware of its effectiveness. However, just because the product is on the
market doesn't mean consumers are aware of its effectiveness. The analysts
conclusion states that as consumers learn about tongue scrapers, the market for
breath freshening products will decline, regardless of whether the tongue scraper
has already been on the market.
(C) While this certainly doesnt help the market analysts case, we dont know
what percentage of dentists recommend flossing over the tongue scraper, and
we cant be sure how this recommendation affects the consumption of breath
freshening products.
(D) CORRECT. This statement weakens the argument by attacking one of the
main assumptions of the argument: people who use breath freshening products
dont already know about the effectiveness of the tongue scraper. If 94% of those
who consume breath freshening products already know about the tongue
scraper, and if these consumers have continued to purchase breath freshening
products, then only 6% of those who consume breath freshening products could
decide to stop purchasing these products upon learning about the tongue
scrapers effectiveness in fighting bad breath. Even if all 6% stopped purchasing
the products, this would hardly create a significant decline in the market.
(E) This statement could weaken the argument by attacking the first assumption
(consumers are primarily interested in products that are most effective in fighting
bad breath). These particular consumers would not necessarily stop consuming
breath freshening products upon learning of a more effective product, such as
the tongue scraper. However, we have no information on what percentage of the
market these people represent.

18. Most water companies in the United States add fluoride to tap water to help prevent cavities. Some dentists argue,
however, that this practice actually causes more harm than good because people overestimate the protection
afforded by the fluoride and do not take the proper steps to care for their teeth, such as brushing and flossing after
every meal. If water companies did not add fluoride, the dentists claim, people would be forced to be more active
in their dental hygiene and tooth decay would decline as a result.

JUST OPPOSE THE CONLUSION .............FOR WEAKEN ....PEPOPLE WILL NOT BE FORCED TO BE MORE
ACTIVE AND TOOTH DECAY WILL NOT DECLINE ..WHY ..WHY WILL THEY NOT BE MORE ACTIVE AND WHY
WILL THE DECAY NOT DECLINE .........BECAUSE THEY ARE UNAWARE OF THE FACT ................FLUORINE
IS BEING ADDED

Which of the following, if true, most weakens the


dentists claims?
Fluoride is widely used in commercially available dental care products.
Most Americans are not aware that fluoride is added to tap water.
Annual visits to the dentist are the most effective means of controlling tooth decay.
The United States has the lowest rate of tooth decay in the world.
Most Americans already brush their teeth daily

18.
The dentists argue that adding fluoride to tap water lulls people into a false sense
of dental security because they rely too heavily on the fluoride to do work they
should do for themselves. The dentists rely on the assumption that people are
aware that fluoride is added to the water. The correct answer will weaken the
conclusion by contradicting this assumption.
(A) The ingredients in commercially available dental care products are irrelevant
to the dentists argument about whether adding fluoride to tap water is ultimately
beneficial.
(B) CORRECT. If most Americans are not aware that fluoride is added to tap
water, then they must not be relying on it to protect their teeth. Poor dental
hygiene might just be due to laziness or apathy, and this behavior would remain
unchanged even if the water companies stopped adding fluoride to tap water. If
anything, tooth decay would be likely to increase, since people would no longer
get any protection from the tap water.
(C) The most effective means of controlling tooth decay, whether by annual
dental exams or some other means, is irrelevant to the dentists argument about
whether adding fluoride to tap water is ultimately beneficial.
(D) How the United States ranks in terms of tooth decay is irrelevant to the
dentists argument about whether adding fluoride to tap water is ultimately
beneficial.
(E) The current dental hygiene routine of most Americans is irrelevant to the
dentists argument about whether adding fluoride to tap water is ultimately
beneficial.

22. The anticipated retirement of tens of thousands of baby boomers will create an unprecedented opportunity to
move significant numbers of people into career-track jobs at family-supporting incomes. Major industries, from
health care and construction to automotive repair, will soon face deep shortages of workers as a result of projected
growth and boomer retirements. Fortunately, many of these jobs have relatively low barriers to entry and could be
filled by out-of-work young people. To achieve this result, the city government should convene employers and
educators to determine how best to create paths of upward mobility in these fields.

THIS RESULT WILL NOT ACHEIEVED ....VY THE CITY GOVERNEMNT CONVENING EMPLOYERS
AND EDUCATORS ........WHY WHY WILL CITY GOVERNMENT NOT BE ABLE TO ACHEIVE THE
RESULTS BY CONVENING EMPLOYERS AND EDUCATORS TO DETERMINE HOW BEST T CREATE
PATHS OF UPWARDS MOBILITY

Which of the following, if true,


most weakens the argument?

Immigration reform will limit the pool of available workers.


Government efforts have been shown to affect employment trends only rarely.
The best available positions require skills not possessed by the vast majority of the unemployed.
A small proportion of baby boomers will not retire as soon as is anticipated.
Many out-of-work young people are unaware of these looming employment opportunities

23. According to a recent research study, more than 90% percent of graduates of private high schools in a certain
county continue their education in college. By contrast, only 65% of graduates of public high schools subsequently
pursue college education. Therefore, if parents in the county wish to increase the likelihood that their children will
attend college, they should send them to private rather than public schools. Which of the following statements
would most seriously weaken the argument above?
Graduates of private schools typically score higher on standardized tests and other tests of academic achievement.
While private schools are typically very expensive, attendance of public school is free for the residents of the
county.
In comparison with graduates of private schools, a substantially greater proportion of public school graduates
receive need-based financial aid for their college education.
In comparison with private schools, public schools provide more opportunities for student involvement in sports
and other athletic activities, which almost always increase the likelihood of students acceptance to colleges.
Since most public schools are located in rural areas of the county populated primarily by farmers, nearly 30% of
students from public high schools choose to pursue farming occupations rather than apply to colleges

23.
The argument concludes that children are more likely to attend college if they are
sent to private high schools instead of public high schools. The basis for this
claim is the higher percentage of graduates of private schools pursuing college
education. It is assumed that public schools are inferior to private schools as a
training ground for college. Any statement that provides an alternate explanation
for the fact that public school graduates attend college at lower rates than private
school graduates would weaken the argument.
Another way to interpret this question involves the concepts of correlation and
causation. The arguments premise states that private school attendance (vs.
public school attendance) is highly correlated with college attendance. The
conclusion of the argument is essentially that private school attendance CAUSES
college attendance (and therefore, parents ought to send their children to private
schools to ensure eventual college attendance). This conclusion depends on the
assumption that the causation does NOT work the other way in other words,
that readiness or desire to attend college does not influence the choice of public
or private school. Any evidence that readiness or desire to attend college DOES
influence the choice of public or private school will weaken the argument.
(A) While higher test scores might increase students' chances of admission to
college, they are unrelated to whether students will actually attend college. Even
if one could prove that earning higher test scores makes a student more likely to
attend college, this statement would not weaken the argument, but rather
strengthen it.
(B) Since the conclusion centers on the likelihood of attending college, economic
and financial considerations are outside the scope of the argument.
(C) Since the amount of need-based aid is not directly related to whether a
student will attend college, this statement is outside the scope of the argument.
(D) While better athletic opportunities could increase students chances of
admission to college, they are unrelated to whether students will actually attend
college. Also, even though the advantages of public school mentioned in this
statement were taken into account by the study, the proportion of graduates of
public schools attending colleges remains substantially lower than the proportion
of graduates of private schools.
(E) CORRECT. This answer choice demonstrates that the difference in the
percentage of graduates attending colleges stems not from any advantage
provided by private schools but from the fact that a subset of the graduates of
public high schools simply choose to pursue a different career path. In other
words, 30% of the graduates of public schools voluntarily choose not to pursue a
college education. Yet 65% - out of the 70% of graduates remaining - end up in
colleges. This statement indicates extremely high college matriculation rates for
students who want to attend college after graduation from public high schools.
Using the concepts of correlation and causation, this answer choice provides the
alternative causation for the correlation observed. In other words, "desire to
attend college" is ALREADY lower in the rural areas where public schools
happen to be located. According to this evidence, attendance at private or public
school is the effect, not the cause, of "desire to attend college."

25. The current administration and Congress have once again practiced bad public policy in failing to increase Pell
grants or at least limit their reduction for next years budget. Pell grants improve access to higher education for
those who have historically been disadvantaged in our society by financial or other life circumstances, thereby
helping recipients elevate themselves to the middle class. Without that access, the gap between the rich and poor
in this country will continue to widen, increasingly straining the stability of our democracy. Which of the following,
if true, most seriously weakens the conclusion of this argument?

UR TARGET IS TO IDENTIFY THE RIGHT CONCLUSION AND ITS BAISS .............NOW KEEPING THE BAISS INTACT IN ANY CASE REVERSE
THE CONSLUION AND ASK WHY ...IS OPPSOTE OF CONLUSION TRUEEE..WHY ...U GET THE ANSWER

Total spending on programs targeted at improving access to higher education for disadvantaged students will
increase in next years federal budget.
The neediest candidates for Pell grants often lack information about their eligibility for such grants.
Congress recently authorized a bill that will increase after-school programs in urban communities.
On average, an individual Pell grant funds less than 15% of the full cost of attending a four-year college or
university.
Federal spending on education for next year will increase as a percentage of the total budget.

25.
The conclusion is that the government has practiced bad public policy in failing
to increase Pell grants or at least limit their reduction for next year's budget." The
basis for that claim is that Pell grants improve access to higher education, which
allows lower-income students to improve their economic standing. The main
assumption this argument relies on is that Pell grants are the only means
available to lower-income students who wish to access higher education. The
correct answer will weaken the conclusion by contradicting this assumption.
(A) CORRECT. If total spending on access to higher education will increase, then
the federal government has addressed the issue that the author cites, albeit
through means other than Pell grants.
(B) Whether candidates for Pell grants are aware of their eligibility is irrelevant to
the claim that the government has practiced bad public policy.
(C) This choice may sound like a counterargument (that Congress is somehow
practicing good public policy by authorizing a bill that will increase after-school
programs in urban communities) to the argument presented (that the government
is practicing bad public policy by failing to safeguard Pell grants). However, we
have no evidence that after-school programs in urban communities help lowincome
students afford higher education, so this does not weaken the argument
presented by the author.
(D) The dollar amount of the Pell grants is irrelevant. To this argument, it matters
only that they provide some help at all.
(E) Increased spending on education as a percentage of the total budget does
not necessarily imply that low-income students will have better access to higher
education. In fact, it does not even imply that education spending (in dollars) will
increase.

27. Joe: The sign on this garden says Do Not Pick The Flowers.
Sally: You are right. But there are a lot of flowers here. Picking just one will not hurt.
Joe: That is not true. If everyone thought that way and picked a flower, the garden would be destroyed.
Joes response to Sally is questionable because ______.
it ignores the possibility that there may be circumstance where destroying the garden is justified
it is circular in reasoning
it contradicts itself by pointing out that collective action has a different result than does an individual action
it cites the consequence of everyone performing an action rather than that of Sallys action itself
it attacks Sallys character in order to undermine her credibility
27.
Sally makes a claim that picking one flower will not cause any harm. Joe replies
that her claim is not true and supports his response by citing the consequence of
everyone picking a flower. By doing so, Joe attempts to shift the focus away from
Sally's actual claim.
(A) Whether there are circumstances under which destroying the garden is
justified is irrelevant to Sallys claim that picking one flower is not harmful and
Joes argument refuting that claim.
(B) A circular argument assumes that which it is trying to prove. The following is
a circular argument: "Only an untrustworthy person would run for office. The fact
that politicians are untrustworthy is proof of this." Joes argument is not circular:
Joe does not rely on the assumption that Sallys statement is not true in order to
argue that Sallys statement is not true.
(C) While Joe does point out that the consequence of the collective action is
different from that of Sallys individual action, his argument is questionable
because in doing so he attempts to shift the focus away from Sallys actual claim,
not because he contradicts himself. There is no contradiction here.
(D) CORRECT. Joe attempts to refute Sallys claim by asserting that the
collective action of everyone would destroy the garden. This argument is
questionable because Sally merely made a claim about the consequence of
picking just one flower, not about the consequences of everyone doing so. By
using the consequences of everyone picking a flower to refute Sally's claim on
the consequences on picking just one flower, Joe implies that Sally's picking of
the one flower will necessarily lead to everone picking a flower (hence, leading to
the destruction of the garden). Not only is this assumption not necessarily true,
but it is also not supported by anything Joe says in his response: he merely
states a premise based on a hypothetical ("If everyone thought that way and
picked a flower...."). This questionable argumentive technique of shifting the
focus from the consequence of a single action to the consequence of a much
larger collective action without proving a cause-and-effect link between the single
and collective actions is known as the "fallacy of the slippery slope assumption."
(E) Joe says nothing that attacks Sallys character.

Re: Wide dissemination of wireless networks


Sat Mar 05, 2011 6:42 am

shantascherla wrote:Wide dissemination of wireless networks in cities is a practical way to meet the needs of city households, schools and businesses.
Rural communities have found that wireless networks are both more reliable and cheaper than land-based networks.

Which of the following would most likely be cited by a supporter of the argument?

Urban areas do not pose additional problems for the effective operation of wireless networks.
Wireless networks work far better where population density is low.
Iceland, a very rural country, successfully uses wireless networks.
The expenses of wireless transmission in areas with large buildings is much higher.
Poor neighborhoods have less access to cable internet than do educators or businesses

The answer is A.
In my analysis the most support for the argument "rural communities have found that wireless networks are both more reliable and cheaper than land-
based networks" comes from b rather than a

Can somebody please explain this

shanta,
The conclusion of this argument is: Wide dissemination of wireless networks in cities is a practical way to meet the needs of city households, schools
and businesses.
The premise of the argument is: Rural communities have found that wireless networks are both more reliable and cheaper than land-based networks.

Thus, we are concluding that wireless networks will meet the needs of cities because they have met the needs of rural communities. The assumption we
must make is that rural areas and cities are comparable in terms of our ability to implement these networks and make them work.

We are asked to choose an answer choice that strengthens the argument. A supporter of the argument will affirm this assumption. Answer choice A
does this nicely.

Answer choice B actually goes AGAINST the conclusion--it says that wireless networks work much better in areas where population is low. Thus, it
casts doubt on the idea that wireless networks will be a good solution for cities.
Jamie Nelson
ManhattanGMAT Instructor
20. Some animals, such as dolphins, dogs, and African grey parrots, seem to exhibit cognitive functions typically
associated with higher-order primates such as chimpanzees, gorillas, and humans. Some parrots, for example,
have vocabularies of hundreds of words that they can string together in a comprehensible syntax. This clearly
shows that humans and primates are not the only animals capable of using language to communicate. One parrot,
named Alex, has been known to ask to be petted or kissed and will exhibit aggression if the gesture offered is not
the specific one requested.

Which of the following, if true, would most strengthen the conclusion above?

Dolphins can be trained to assist divers in ocean rescues.


Gorillas in captivity often learn hand signals for food and water.
Dogs are capable of sensing their owners moods and often exhibit concern if they sense sadness.
Chimpanzees can memorize long sequences of key punches on machines that dispense food.
Alex does not exhibit aggression when offered a gesture that he specifically requested.

20.
The conclusion of the argument is that humans and primates are not the only
animals capable of communicating with language. The basis for this claim is that
a parrot named Alex becomes upset when he is not given the gesture he verbally
requests. We are asked to strengthen the claim.
(A) The conclusion is about non-primates or non-humans being able to use
language to communicate. Assisting divers in ocean rescues is not relevant.
(B) The conclusion is about non-primates or non-humans being able to use
language to communicate. Gorillas are primates, as stated in the first sentence
of the argument.
(C) Sensing the mood of ones owner and exhibiting concern is not a form of
language communication.
(D) The conclusion is about non-primates or non-humans being able to use
language to communicate. Chimpanzees are primates, as stated in the first
sentence of the argument.
(E) CORRECT. If Alex does not exhibit aggression when offered a gesture that
he specifically requested, it suggests that Alex can tell the difference between the
gestures that he requests and those that he does not. In other words, he is a
non-primate / non-human but he is communicating via language. If he also
exhibited aggression when offered the gestures he requested, it would be more
difficult to claim that he was communicating via language.

Alex is communicating with humans of course, probably its master.

Option E is correct because among other things it's the only option that talks about communication.

Options B & D can be left out since chimps and gorillas come under the primate categories according to the argument.

Among the other three options, in C dogs are sensing or perceiving which is not the same thing as communicating.

In A dolphins are being trained but training is a one-way process whereas communication is a two-way process.

In option E suppose Alex asks you to pet him and no matter whether you pet him or not he gets aggressive, then it is not communication. But the option
E, together with the evidence in the argument implies that he only gets aggressive when he doesn't get the desired response. This means that he can
communicate and also comprehend the response - a two-way process.

Ideally you shouldn't have to think this far - the fact that none of the other options involve communication except E should get you to the answer.

Cheers!

19.
The conclusion is that medical schools are misguided. The basis for this claim is
that they pay little attention to preventive medicine. The argument would be
made stronger by a statement concerning the benefits of preventive medicine.
(A) CORRECT. Vaccines are a type of preventive medicine that have known
benefits, i.e., preventing contagious diseases.
(B) This statement supports the idea that the more time and money are spent on
curative medicine than on preventive medicine. However, it does not speak to
the benefits of preventive medicine.
(C) The number of students enrolled in medical school has nothing to do with the
benefits of preventive medicine.
(D) Accidental causes have nothing to do with the benefits of preventive
medicine.
(E) The number of doctors in certain specialties has nothing to do with the
benefits of preventative medicine.

19. Medical education in the United States has focused almost exclusively on curative medicine, while preventive care
has been given scant attention. This is misguided. Medical schools should invest as much time in teaching their
students how to prevent illness as in teaching them how to cure it.
CATCH THE OPINION OF THE AUHTOR AS CONCLUSION...............AND HOLD IT .....AND THEN ASK WHYY ...........FROM THE COMPLETE
OPINION NOT JUST A PART AND U MIND WILL TELL U THE ANSWWER....
BT LET UR MIND REPLY .......WHY ..AND ITS DONE

.ALSO WHILE READING THE OPTIONS U AN THEN GET IT

Which of the following, if true, most strengthens


the argument above?

Many contagious diseases can be prevented with vaccines.


In 1988, for every three cents the United States spent on prevention, it spent 97 cents on curative treatment.
The number of students enrolled in medical school is the highest it has ever been.
More people die each year from disease than from accidental causes.
As the population grows, the number of doctors in certain specialties has not been keeping pace.
Hi mates,

uf! I don't see any clear answer...

I'd go with A

What's the conclusion? That medical schools should invest more in teaching to prevent diseases than to cure them. Well, A supports the argument
by explaining that some diseases can be prevented

OA and Source?

30. When a company refuses to allow other companies to produce patented technology, the consumer invariably
loses. The company that holds the patent can charge exorbitant prices because there is no direct competition.
When the patent expires, other companies are free to manufacture the technology and prices fall. Companies
should therefore allow other manufacturers to license patented technology.

The argument above presupposes


which of the following?

Companies cannot find legal ways to produce technology similar to patented technology.
Companies have an obligation to act in the best interest of the consumer .
Too many patents are granted to companies that are unwilling to share them.
The consumer can tell the difference between patented technology and inferior imitations.
Consumers care more about price than about quality

30.
The conclusion of the argument is that companies should allow other
manufacturers to license patented technology. The basis for that claim is that not
doing so keeps prices high and harms the consumer. We're asked what the
argument assumes ("presupposes") in drawing its conclusion. The correct
answer will fill the logic gap between the idea that keeping prices high harms the
consumer and that companies should allow other manufacturers to license
patented technology. The conclusion is based on the assumption that
companies have an obligation of some kind to do what's best for the consumer.
(A) This does not address the moral obligation to the consumers (i.e. should) of
the companies who produced the patented technology, the main point of the
conclusion. Furthremore, even if companies could find legal ways to produce
similar technologies, the patented technology could still command exorbitant
prices, thereby harming the consumer.
(B) CORRECT. The conclusion only makes sense if companies have an
obligation to act in the best interest of the customer, as this choice states.
(C) This generally follows along with the author's claim, but we are not required
to assume this in order to reach the conclusion that companies who are granted
patents are obligated to look out for the best interests of their customers.
(D) This addresses a tangential issue of whether or not consumers could notice
the difference between a new patented technology and a possible imitation. This
does not address the core issue of the obligation to the consumer.
(E) This does not address the obligation of the companies toward the consumers,
or indeed the companies at all.

30. When a company refuses to allow other companies to produce patented technology, the consumer invariably
loses. The company that holds the patent can charge exorbitant prices because there is no direct competition.
When the patent expires, other companies are free to manufacture the technology and prices fall. Companies
should therefore allow other manufacturers to license patented technology. The argument above presupposes
which of the following?
Companies cannot find legal ways to produce technology similar to patented technology.
Companies have an obligation to act in the best interest of the consumer.
Too many patents are granted to companies that are unwilling to share them.
The consumer can tell the difference between patented technology and inferior imitations.
Consumers care more about price than about quality

It is a tricky technique.

Conc: comp should allow other comps to license pat.tech

Prem: comp w/pat monopoly charges lots of $; cons pays more. Cons would pay less if other comps could use tech.

To negate "Companies CANNOT <do something>" we would say "companies COULD, at least some of the time, <do whatever that thing is>"
The assumption "exists" between one of the premises and the conclusion, so we're basically testing the connection between the two.

The correct answer should make a connection between the two. If we negate the correct answer, it should actually mess up the connection.
The connection is the key thing - not just the conclusion (or just the premise).

P: If there's a monopoly, the consumer pays more.


A: (other) companies can't mimic the tech in some other way.
LEN: (other) companies can sometimes mimic the tech in some other way.
C: companies should license their tech to other comps.

Does the A help us draw the conclusion from P? Does the LEN hurt the conclusion we're drawing from P?

In this case, not really. This choice doesn't actually help us connect P to C.

Try this:
P: If there's a monopoly, the consumer pays more.
A: Companies must act in the best interest of the cons.
LEN: Comps don't have to act in the best interest of the cons.
C: companies should license their tech to other comps.

Does the A help us draw the conclusion from P? Does the LEN hurt the conclusion we're drawing from P?

In this case, bingo! The A does actually help connect the P to the C. And the LEN does actually mess up that connection.

Joe: Company X has a monopoly, so I have to pay more to buy that product!
Susie: Companies don't have to set their policies to make sure you don't have to pay as much money. They don't have to care about your best interests if
they don't want to.
Joe: Exactly! Therefore, X should license its tech to other companies so that I can pay less money!

Hmm. No. Susie's comment totally kills Joe's argument. :) What if Susie had said, "I know, right? Companies totally have an obligation to make sure
they set prices based upon their customers' best interests!" That would help Joe's argument.
Stacey Koprince
Instructor
Director of Online Community
ManhattanGMAT

30. When a company refuses to allow other companies to produce patented technology, the consumer invariably
loses. The company that holds the patent can charge exorbitant prices because there is no direct competition.
When the patent expires, other companies are free to manufacture the technology and prices fall. Companies
should therefore allow other manufacturers to license patented technology. The argument above presupposes
which of the following?
Companies cannot find legal ways to produce technology similar to patented technology.
Companies have an obligation to act in the best interest of the consumer.
Too many patents are granted to companies that are unwilling to share them.
The consumer can tell the difference between patented technology and inferior imitations.
Consumers care more about price than about quality.

It is a tricky technique.

Conc: comp should allow other comps to license pat.tech

Prem: comp w/pat monopoly charges lots of $; cons pays more. Cons would pay less if other comps could use tech.

To negate "Companies CANNOT <do something>" we would say "companies COULD, at least some of the time, <do whatever that thing is>"

The assumption "exists" between one of the premises and the conclusion, so we're basically testing the connection between the two.

The correct answer should make a connection between the two. If we negate the correct answer, it should actually mess up the connection.
The connection is the key thing - not just the conclusion (or just the premise).

P: If there's a monopoly, the consumer pays more.


A: (other) companies can't mimic the tech in some other way.
LEN: (other) companies can sometimes mimic the tech in some other way.
C: companies should license their tech to other comps.

Does the A help us draw the conclusion from P? Does the LEN hurt the conclusion we're drawing from P?

In this case, not really. This choice doesn't actually help us connect P to C.

Try this:
P: If there's a monopoly, the consumer pays more.
A: Companies must act in the best interest of the cons.
LEN: Comps don't have to act in the best interest of the cons.
C: companies should license their tech to other comps.

Does the A help us draw the conclusion from P? Does the LEN hurt the conclusion we're drawing from P?
In this case, bingo! The A does actually help connect the P to the C. And the LEN does actually mess up that connection.

Joe: Company X has a monopoly, so I have to pay more to buy that product!
Susie: Companies don't have to set their policies to make sure you don't have to pay as much money. They don't have to care about your best interests if
they don't want to.
Joe: Exactly! Therefore, X should license its tech to other companies so that I can pay less money!

Hmm. No. Susie's comment totally kills Joe's argument. :) What if Susie had said, "I know, right? Companies totally have an obligation to make sure
they set prices based upon their customers' best interests!" That would help Joe's argument.
Stacey Koprince
Instructor
Director of Online Community
ManhattanGMAT
MAT/MBA EXPERT
ChessWriter wrote:
When a company refuses to allow other companies to produce patented technology, the consumer invariably loses. The company that holds the patent
can charge exorbitant prices because there is no direct competition. When the patent expires, other companies are free to manufacture the technology
and prices fall. Companies should therefore allow other manufacturers to license patented technology.

The argument above presupposes which of the following?

(A) Companies cannot find legal ways to produce technology similar to patented technology.
(B) Companies have an obligation to act in the best interest of the consumer.
(C) Too many patents are granted to companies that are unwilling to share them.
(D) The consumer can tell the difference between patented technology and inferior imitations.
(E) Consumers care more about price than about quality.

The premise is about X: The CONSUMER loses when a company refuses to allow other companies to produce patented technology.
The conclusion is about Y: COMPANIES should therefore allow other manufacturers to license patented technology.

The assumption is that X is linked to Y: that, because the CONSUMER loses, COMPANIES should change what they are doing -- even though these
companies can charge EXORBITANT prices. Why should a company that can charge exorbitant prices change its business model? What's bad for the
consumer clearly is GOOD for the company.

Answer choice B exposes the assumption:

For the conclusion here to be valid, it MUST BE TRUE THAT companies have an obligation to act in the best interest of the consumer.

If this answer choice is negated -- if companies DO NOT have an obligation to act in the best interest of the consumer -- then they have no reason to
license patented technology, invalidating the conclusion of the passage.

The correct answer is B.


ChessWriter wrote:
When a company refuses to allow other companies to produce patented technology, the consumer invariably loses. The company that holds the patent
can charge exorbitant prices because there is no direct competition. When the patent expires, other companies are free to manufacture the technology
and prices fall. Companies should therefore allow other manufacturers to license patented technology.

The argument above presupposes which of the following?

(A) Companies cannot find legal ways to produce technology similar to patented technology.
(B) Companies have an obligation to act in the best interest of the consumer.
(C) Too many patents are granted to companies that are unwilling to share them.
(D) The consumer can tell the difference between patented technology and inferior imitations.
(E) Consumers care more about price than about quality.

The premise is about X: The CONSUMER loses when a company refuses to allow other companies to produce patented technology.
The conclusion is about Y: COMPANIES should therefore allow other manufacturers to license patented technology.

The assumption is that X is linked to Y: that, because the CONSUMER loses, COMPANIES should change what they are doing -- even though these
companies can charge EXORBITANT prices. Why should a company that can charge exorbitant prices change its business model? What's bad for the
consumer clearly is GOOD for the company.

Answer choice B exposes the assumption:

For the conclusion here to be valid, it MUST BE TRUE THAT companies have an obligation to act in the best interest of the consumer.

If this answer choice is negated -- if companies DO NOT have an obligation to act in the best interest of the consumer -- then they have no reason to
license patented technology, invalidating the conclusion of the passage.

The correct answer is B.


Thanks Mitch, that is indeed the Correct Answer. It makes a lot of sense now

Quote
Below are my negations of all the answer options. I have negated all statements mainly for my own practise. But, I am still confused between options
A and B

Although, I do understand why B is an underlying assumption for the stimulus to be true, I still do not get why it is a stronger contender for being the
right answer than Option A.

If companies can indeed find legal ways to produce technology similiar to patented technology, then there would be no point forcing them to do as the
stimulus suggests. So, Isn't Option A a stronger contender for being the right answer than Option B.

(A) Companies cannot find legal ways to produce technology similar to patented technology.
Negation: Companies CAN find legal ways to produce technology similar to patented technology.

(B) Companies have an obligation to act in the best interest of the consumer.
Negation: Companies DO NOT have an obligation to act in the best interest of the consumer.

(C) Too many patents are granted to companies that are unwilling to share them.
Negation: Too FEW patents are granted to companies that are unwilling to share them.

(D) The consumer can tell the difference between patented technology and inferior imitations .
Negation: The consumer can NOT ALWAYS tell the difference between patented technology and inferior imitations.

(E) Consumers care more about price than about quality.


Negation: Consumers DON"T care more about price than about quality.
[/quote]tell the difference between patented technology and inferior imitations.

(E) Consumers care more about price than about quality.


Consumers DON"T care more about price than about quality.
[/quote]

ChessWriter wrote:
Below are my negations of all the answer options. I have negated all statements mainly for my own practise. But, I am still confused between options
A and B

Although, I do understand why B is an underlying assumption for the stimulus to be true, I still do not get why it is a stronger contender for being the
right answer than Option A.

If companies can indeed find legal ways to produce technology similiar to patented technology, then there would be no point forcing them to do as the
stimulus suggests. So, Isn't Option A a stronger contender for being the right answer than Option B.

(A) Companies cannot find legal ways to produce technology similar to patented technology.
Negation: Companies CAN find legal ways to produce technology similar to patented technology.

The negation of A -- that companies can find legal ways to produce similar technology -- contradicts the PREMISE:
The company that holds the patent can charge exorbitant prices because THERE IS NO DIRECT COMPETITION .
When the correct answer is negated, it cannot contradict the premise.

Answer choice A simply REAFFIRMS the premise: if companies cannot find legal ways to produce similar technology, then THERE IS NO DIRECT
COMPETITION.
The correct answer -- the ASSUMPTION -- is what must be true in order to LINK this premise to the conclusion.

27. Recent research has indicated that married people are not only
happier than unmarried people, but also healthier.
This study has been widely reported by the media, with most
commentators concluding that being married is good
for ones health and attitude.

The conclusion of the media commentators depends on which of the


following
assumptions?
The longer people are married, the happier and healthier they become.
Married couples who had a large, extravagant wedding are happier than
those who had a small, simple ceremony.
Married people cannot get depressed.
Single people with depression or health problems are just as likely to get
married as are other single people.
Some marriages are more harmonious than others

Research indicates that there is a connection between being married and being
happy and healthy. Media commentators have concluded that marriage causes
happiness and health. However, one could reasonably conclude from the
research that the cause and effect are the reverse: being happy and healthy
makes a person more likely to get married.
(A) The research compared married people to unmarried people. Neither the
researchers nor the media commentators made any distinction between
newlyweds and those who had been married a long time, so this assumption is
not necessary.
(B) The type of wedding is outside the scope of this argument. The research
compared married people to unmarried people, but made no distinction based
upon the type of wedding. Thus, this assumption is unnecessary.
(C) At first, this statement may seem necessaryafter all, if the commentators
conclude that marriage causes happiness, a lack of depression in married people
would certainly support that conclusion. However, the statement is too extreme.
One depressed married person does not invalidate the research indicating that,
on average, married people are healthier and happier than non-married people.

(D) CORRECT. This statement eliminates the alternative interpretation


of the
research findingsthat being happy and healthy makes a person more
likely to
get married.
(E) The research compared married people to unmarried people. Neither the
researchers nor the media commentators made any distinction between
harmonious marriages and combative marriages, so this assumption is not
necessary.

4. Scientist: Evolutionary biology has long held that the most attractive males of a species, defined as those with the
highest quality physical traits that have no Darwinian survival value, will draw the most female mates. The
resulting male offspring will inherit that attractiveness and themselves have more children as a
result, thus ensuring widespread dissemination of the grandparents genes. Recently, however, scientists have
found that the sons of high quality male flycatchers failed to inherit the fathers mating status. Further, the most
attractive males were so busy mating that they neglected their offspring; as a result, the sons of homelier
birds, who took better care of their offspring, had more success at propagating the species. The two
portions in boldface play which of the following roles in the scientists argument?
The first is the conclusion of a theory disputed by the scientist; the second is the scientists new contention based
upon the latest evidence.
The first is a premise of a long-held biological theory; the second is an example of how this theory works.
The first is an explanation of how a biological theory is thought to work; the second is an example of research
results that do not support this theory.
The first is an example of a theory that used to be prevalent; the second is the new theory that is now considered
predominant by scientists.
The first introduces a long-held theory that the scientist is going to disprove; the second is the scientists new
theory to replace the one she disproved.
5. Traditionally, video game manufacturers have been most strongly influenced by serious video gamers. Because
devoted gamers have historically purchased the majority of video games, companies react to the desires of this
market segment. Normally, devoted gamers crave speed and action; thus, most manufacturers continue to
produce games with faster chips and flashier graphics. Unfortunately, faster chips and flashier graphics are
no longer in the industrys best interest. The devoted gaming market is deeply stagnant, and it wont soon
expand. To infuse new life into the video game market, manufacturers must simplify the functionality of their
games. By doing so, current non-gamers will be attracted to join the ranks of video game fans. In the argument,
the two portions in boldface play which of the following roles?
The first is a situation that the author believes to be true; the second offers evidence to explain this situation.
The first is a situation that the author argues should not continue; the second provides evidence that supports the
authors position.
The first is a statement of fact that contradicts the authors position; the second is the authors position.
The first is a statement of fact that supports the authors position; the second is a consideration that weighs
against the authors position.
The first is a prediction that the author believes should not hold in this case; the second is an assumption that
weighs against the authors position.

bold statement does not introduce a new theory; it simply presents


research results that contradict the original theory.
5.
The author explains that devoted gamers traditionally dictate the design of video
games. However, due to changes in the market, the author argues that this
system is no longer in the best interest of the industry. Instead, to infuse new life
into the video game market, manufacturers should simplify their games in order
to attract non-gamers into the gaming fold.
(A) The first boldface portion does relate a situation that the author believes to be
true. The second boldface portion, however, does not explain this situation;
instead, it offers evidence to demonstrate why this situation should not continue.
(B) CORRECT. The first boldface portion is a situation that the author believes to
be true now. Due to changes in the market, however, the author believes this
situation should not continue. The second boldface portion provides evidence to
support the authors contention that the best way to grow the gaming market is to
attract new gamers.
(C) The first boldface portion is a statement of fact that contradicts the authors
position. The second boldface portion, however, provides evidence to support the
authors position, but it is not the position itself.
(D) The first boldface portion is a statement of fact that contradicts the authors
position. The second boldface portion provides evidence to support the authors
position.
(E) The first boldface portion is not a prediction; rather, it is a statement of fact
(or description of a situation) that the author believes should not hold in this case.
The second statement is not an assumption, nor does it weigh against the
author's position; instead, it is a premise that provides evidence in support of the
authors position.

7. The city government should invest surplus funds in improving the citys transportation network. Most of the
network was put in place at a time when the city was much smaller in both area and population. The
subway system is outdated and understaffed. The buses rarely run on schedule and their routes are inconvenient.
If the city does not make changes soon to the network, it will see many of its prized industries
relocate to more convenient cities and, as a result, the citys financial health will be jeopardized. In
the argument above, the two portions in boldface play which of the following roles?
The first is an explanation of a current state of affairs; the second is a prediction based on that state of affairs.
The first is a statement of fact in opposition to the authors conclusion; the second is that conclusion.
The first emphasizes an existing problem; the second offers a proposal to solve that problem.
The first is information the author suggests has been overlooked in the situation at hand; the second describes
that situation.
The first is a justification of an impending problem; the second describes the consequences of that problem.

7.
The conclusion of the argument is that the city will see many of its prized
industries relocate to more convenient cities and the city's financial health will be
jeopardized if the city does not make changes soon to the transportation network.
This is also the second bolded sentence. The first bolded sentence states that
most of the network was put in place at a time when the city was much smaller in
both area and population. We need to find a choice that correctly describes both
of these bolded statements.
(A) CORRECT. This choice states that the first statement is an explanation of a
current state of affairs. This explanation is consistent with the passage. The
answer choice goes on to describe the second bolded statement as a prediction
based on that state of affairs. This is also consistent with the passage: the
second bolded statement predicts what will happen as a result of the inadequacy
of the current transportation network.
(B) The first statement is indeed a statement of fact, but the author cites it in
order to bolster his or her claim; thus the statement is not in opposition to the
conclusion. The second statement is the conclusion of the argument.
(C) The first statement does not "emphasize an existing problem" but rather
explains that existing problem (of an overtaxed subway). Moreover, the second
statement does not "offer a proposal to solve that problem" but rather warns of
what will happen if the problem is not solved.
(D) The first statement arguably presents information that "the author suggests
has been overlooked in the situation at hand"; however, the second statement
does not describe that situation, but rather proposes a hypothetical outcome in
the future.
(E) The first statement is not really "justification" (a term that implies approval on
the part of the author) but rather an "explanation"; nor does it refer to an
"impending problem" but rather an existing problem. Also, the second statement
does not describe "consequences" exactly but rather "potential consequences" if
the problem is left unchecked.
The correct answer is A.

10. United Energy recently invested in a series of large windmills which are able to produce renewable energy with
minimal negative effect to the environment. The company has not drilled oil wells in the same area, even though
greater revenues and profits could be generated from oil wells. Because any drilling would disrupt the
native habitat of certain marine species in the area, some environmentalists assert that, by foregoing this drilling,
United Energy has established that it places environmental impact over financial returns. However,
United Energy may be acting in a manner consistent with its financial goals. Recent patterns of increasing annual
hurricane activity have some experts questioning the long-term viability and profitability of oil wells in the area.
The two boldfaced portions play which of the following roles?
The first supports the conclusion of the argument; the second calls that conclusion into question.
The first states the conclusion of the argument; the second supports that conclusion.
The first supports the environmentalists conclusion; the second states that conclusion.
The first states the environmentalists conclusion; the second provides a consideration in support of that conclusion.
The first supports the conclusion of the argument; the second also supports the conclusion of the argument.

10.
The argument concludes that United Energy may be acting in a manner
consistent with reaching its financial goals. However, this conclusion must be
distinguished from the assertion of the environmentalists described in the
question - that the actions taken by United Energy indicate that the company is
putting environmental concerns ahead of financial returns. The answer choices
may consider either the author's conclusion (which is considered the overall
conclusion) or the conclusion asserted by the environmentalists (which is
considered the opposing opinion).
This is an Analyze the Argument Structure question. The best approach is to
consider each boldface in turn.
(A) The first boldface does not support the author's conclusion that United
Energy may be acting in a manner consistent with its financial goals; the second
does not call the conclusion into question as much as state its opposite.
(B) The first boldface does not state the conclusion of the argument; the second
boldface does not support the conclusion of the main argument, which is that
United Energy may be acting in a manner consistent with its financial goals.
(C) CORRECT. The first boldface supports the environmentalists conclusion that
United Energy is acting in a manner that places environmental impact ahead of
financial returns. The second boldface states this conclusion.
(D) The first boldface is a relationship that does support the environmentalists'
conclusion; however, the second states this conclusion, and does not undermine
it.
(E) The first boldface does not support the author's conclusion, which is that
United Energy may be acting in its financial interest; the second boldface also
does not support this conclusion.
31. Professor Jones has claimed that chemical compound chlorocetin contained in industrial
waste and previously
considered harmless is in fact very dangerous. Jones has examined several areas with high
concentration of
chlorocetin and found that certain vital biological processes, such as photosynthesis, are
slower in
these areas than is usual for the inspected species. The professor says that although he
failed to establish
an exact mechanism by which chlorocetin hampers photosynthesis, his findings are
sufficient to state that chlorocetin
is a dangerous chemical affecting natural world. This conclusion, however, is unwarranted
because all that
Professor Jones actually established is a mere correlation between the level of chlorocetin
and the
speed of photosynthesis correlation that might not reflect any causal relationship between
the factors. The
highlighted portions of the text perform which of the following functions in the argument?
A. the first is the position the author disagrees with while the second is the authors own
position
B. the first contains the argument the author critiques while the second identifies a logical
flaw in this argument
C. the first is an evidence in the argument the author disputes while the second is a
statement supporting the authors
own position
D. the first describes a phenomenon which weighs against the authors opinion while the
second reasons in favor of
this opinion
E. the first contains information that, if true, could overthrow the authors argument while
the second discredits this
information

31. Analysis: There are two persons in the question: Professor Jones and
the author. As one can see, the author does not agree to the argument by
Professor Jones. This conclusion, however, is unwarranted because
(the second boldface) . Therefore, the opinion of the author is very
much at odds with the opinion of Professor Jones. Then, lets take a look
at the first boldface certain vital biological processes, such as
photosynthesis, are slower in these areas than is usual for the
inspected species. This is result of a test or lab or discovery, so it is an
evidence professor Jones used to back up his conclusion Professor
Jones has claimed that....Therefore, answer C is the correct choice. Why
is B incorrect? The first contains the argument the author critiques is
incorrect, because the author does not agree with the evidence (the first
boldface), but only disagrees the conclusion (claim) made by Professor
Jones. We can rule out A, D and E as they are unrelated. B states that
the first contains the argument the author critiques. Nowhere in the
passage does the author critique the argument; he merely attacks the
conclusion that is based on the argument. Furthermore the first sentence
is clearly evidence and not an argument in itself.

29. Nutritionist: Because humans have evolved very little since the development of
agriculture, it is clear that humans
are still biologically adapted to a diet of wild foods, consisting mainly of raw fruits and
vegetables, nuts and
seeds, lean meat, and seafood. Straying from this diet has often resulted in chronic illness
and other physical
problems. Thus, the more our diet consists of wild foods, the healthier we will be. The
boldface portion plays
which one of the following roles in the argument?
A. It is a conclusion for which the only support offered is the claim that straying from a diet
of wild foods has often
resulted in chronic illness and other physical problems.
B. It is a premise for which no justification is provided, but which is used to support the
arguments main conclusion.
C. It is a phenomenon for which the main conclusion of the nutritionists argument is cited as
an explanation.
D. It is an intermediate conclusion for which one claim is offered as support, and which is
used in turn to support the
arguments main conclusion.
E. It is a premise offered in support of the claim that humans have evolved very little since
the development of
agriculture

29. The correct answer choice is (D) The nutritionists argument can be
deconstructed as follows: Premise: Humans have evolved very little since
the development of agriculture. Sub-conclusion/Premise: It is clear that
humans are still biologically adapted to a diet of wild foods, consisting
mainly
of raw fruits and vegetables, nuts and seeds, lean meat, and seafood.
Premise: Straying from this diet has often resulted in chronic illness and
other physical problems. Conclusion: Thus, the more our diet consists of
wild foods, the healthier we will be. The statement referenced in the
question
stem is a sub-conclusion. Answer choice (D), the correct answer,
describes
this role using the phrase intermediate conclusion. This is a great
example
of a question that allows you to accelerate: if you take the correct steps
(fulfill
the Primary Objectives) when analyzing the argument, you already know
the
correct answer and you simply need to scan the answer choices quickly
for a
match. Answer choice (A): This is a Half Right, Half Wrong answer. The
statement is a conclusion, but the only support offered for this conclusion
is
that humans have evolved very little since the development of agriculture.
Since this fact contradicts what is stated in the answer choice, the
answer
choice is incorrect. Answer choice (B): Since we know the statement is a
subconclusion,
there is justification provided and this answer choice is incorrect.
Note that describing the statement as a premise is also accurate, since a
sub-conclusion is a conclusion for one argument and a premise for
another
argument. Answer choice (C): The main conclusion does not explain the
statement referenced in the question stem, so this answer choice is
incorrect.
The answer would be much improved if it said: It is a phenomenon that
helps
explain the main conclusion of the nutritionists argument. Answer
choice
(D): This is the correct answer, and the answer you should have been
looking
for after you analyzed the argument and read the question stem. Answer
choice (E): This is a Reverse answer. The claim that humans have evolved
very little since the development of agriculture is a premise offered in
support
of the statement referenced in the question stem.

28. Seemingly inconsequential changes in sea temperature due to global warming


eventually result in declines in fish
and seabird populations. A rise of just two degrees prevents the vertical mixing of seawater
from different strata.
This restricts the availability of upwelling nutrients to phytoplankton. Since zooplankton,
which feed upon
phytoplankton, feed the rest of the food chain, the declines are inevitable. The boldface
portion plays which
one of the following roles in the argument?
A. It is a hypothesis supported by the fact that phytoplankton feed on upwelling nutrients.
B. It is intended to provide an example of the ways in which the vertical mixing of seawater
affects feeding habits.
C. It helps show how global temperature changes affect larger sea animals directly.
D. It is offered as one reason that global warming must be curtailed.
E. It is offered in support of the idea that global warming poses a threat to all organisms
28. The argument is structured as follows: Premise: A rise of just two
degrees
prevents the vertical mixing of seawater from different strata. Premise:
This
restricts the availability of upwelling nutrients to phytoplankton. Premise:
Zooplankton, which feed upon phytoplankton, feed the rest of the food
chain.
Conclusion: Seemingly inconsequential changes in sea temperature due
to
global warming eventually result in declines in fish and seabird
populations.
The conclusion in the first line is echoed again in the final sentence. The
argument part referenced in the question stem is a premise (note the use
of
the premise indicator since in the last line), and your answer must
indicate
that the role played by the argument part is that of a premise. Answer
choice
(A): The portion referenced in the question stem is not a hypothesis, but
rather a statement of fact. Answer choice (B): The statement referenced
in
the question stem is not an example of the way the mixing of seawater
affects feeding habits, but rather another premise that is then combined
with
the vertical mixing premise to help support the conclusion. Answer choice
(C): This is the correct answer. The phrase it helps show describes a
premise, and in this case the premise is used to support a statement
about
the effect of temperature changes on fish and seabirds. Answer choice
(D):
The argument does not take a position that global warming should be
curtailed. Instead, the argument shows how small changes in sea
temperature lead to population declines, and no opinion of those effects
is
stated. Answer choice (E): This is an Exaggerated Answer. The argument
specifically indicates that fish and seabirds populations will decline. This
answer choices states that all organisms are threatened.
27. Psychologist: The obligation to express gratitude cannot be fulfilled
anonymously. However much society
may have changed over the centuries, human psychology is still driven primarily by personal
interaction. Thus, the
important social function of positively reinforcing those behaviors that have beneficial
consequences for others can
be served only if the benefactor knows the source of the gratitude. The boldface portion
plays which one of the
following roles in the argument?
A. It is an illustration of a premise that is used to support the arguments conclusion.
B. It is used to counter a consideration that might be taken to undermine the arguments
conclusion.
C. It is used to support indirectly a claim that the argument in turn uses to support directly
the conclusion.
D. It is used to identify the social benefit with which the argument is concerned.
E. It is the conclusion that the argument is intended to support.

27. The psychologists argument is structured as follows: Premise:


However
much society may have changed over the centuries, human psychology is
still driven primarily by personal interaction. Sub-conclusion / Premise:
Thus, the important social function of positively reinforcing those
behaviors
[of expressing gratitude] that have beneficial consequences for others
can be
served only if the benefactor knows the source of the gratitude. Main
conclusion: The obligation to express gratitude cannot be fulfilled
anonymously. Often, the identifiers used before the subsidiary
conclusions
are dramatic and somewhat misleading, such as clearly and obviously.
In
these cases, the conclusion is neither clear nor obvious, and those words
are
used to lead the reader into thinking that the conclusion should simply be
accepted without further analysis. The statement referenced in the
question
stem is the main conclusion of the argument, and as the answer choice
correctly describes, the conclusion that the argument is designed to
support.
The argument, when considered in terms of order, is out of order: First
sentence = main conclusion, Second sentence = basic premise, Third
sentence = sub-conclusion. The last sentence is a sub-conclusion, and in
a
Method-AP answer choice the author can describe a sub-conclusion in a
variety of ways: subsidiary conclusion / secondary conclusion /
intermediate
conclusion / supporting conclusion. Note that as predicted, the main
conclusion is not modified by a conclusion indicator but the sub-
conclusion is.
Answer choice (A): The statement in question is not an example of an idea
raised in a premise. Answer choice (B): The answer choice describes a
premise that is used to defend the argument from attack. This would
better
describe the second sentence of the argument. Answer choice (C): This
answer choice describes a premise supporting a sub-conclusion. Again,
this
would better describe the second sentence of the argument. Answer
choice
(D): This answer choice better describes the last sentence. Answer choice
(E): This is the correct answer. By consistently breaking down the
structure of
the argument before reading the answer choices, these problems become
very easy to solve.

Joined: Thu Jan 22, 2009 3:05 pm

Re: In the mid-1920s the Hawthorne Works


Mon Jun 08, 2009 4:55 pm

This is a GPrep Q

Why B over D ?

I eliminated D B/C
working condition's effects on worker's performance ..sounds terrible and is wrong in meaning...
But I like the usage of that in D ..and that has a good referent : experiments ...

Any light on this ?


kramacha1979
Students

Posts: 68
Joined: Thu Jan 22, 2009 3:05 pm

Re: In the mid-1920s the Hawthorne Works


Mon Jun 08, 2009 4:59 pm

Also I liked the use of past tense in D ..investigated..since we are talking about 1920's and have a verb was...
RonPurewal
ManhattanGMAT Staff

Posts: 16635
Joined: Tue Aug 14, 2007 8:23 am

Re: In the mid-1920s the Hawthorne Works


Thu Jun 11, 2009 7:11 am

(b) is the best choice here.

(a) is vague because it's overly indirect: the meaning of "investigate changes ... as to their effects" is unclear. what's more, it's probably considered
unidiomatic as well, at least in this sort of context.

(b) = correct
the participle "investigating" follows "experiments" immediately. no filler words are necessary; this is good concision.
the wording is clear; there are no awkward double possessives, etc., as in some of the other choices.
"would" is used properly here, as a past-tense form of "will". (i.e., if this sentence were translated into the present tense, it would read "...that
changes ... will have")

(c) is ridiculously wordy; there's no way you should give this choice any serious consideration. if you don't realize pretty quickly that this choice is
wrong, you should go back and read through a bunch of correct OG answers, trying to internalize the sights and sounds (the "vibe") of the correct
answers.

(d) "changes in working conditions' effects" is at best awkward and vague, and at worst ambiguous: the intended meaning is the effects of the changes,
but this sentence seems to indicated the effects of the conditions themselves. in other words, a literal reading of this sentence seems to indicate that the
conditions themselves haven't changed - only their effects have. that's not the intended meaning of the original.

(e) "what the effects" is ungrammatical.


also, in constructions of this sort, "what" is generally redundant / unnecessary; it's better merely to say "to investigate X" rather than to say "to
investigate what X is" (or other such wordy construction).
Pueden hacerle preguntas a Ron en castellano
Potete fare domande a Ron in italiano
On peut poser des questions Ron en franais
Voit esitt kysymyksi Ron:lle mys suomeksi

Un bon vtement, c'est un passeport pour le bonheur.


Yves Saint-Laurent
RC - Lyme Disease - GmatPrep
Sat Aug 22, 2009 4:52 am

Lyme disease is caused by a bacterium transmitted to humans by deer ticks. Generally deer ticks pick up the bacterium while in the larval stage from
feeding on infected whitefooted mice. However, certain other species on which the larvae feed do not harbor the bacterium. Therefore, if the population
of these other species were increased, the number of ticks acquiring the bacterium would likely decline.

Which of the following it would be most useful to ascertain in evaluating the argument?

[editor: there were multiple errors in the original transcription, so i've copied and pasted the following answer choices from
another source. in particular, the original transcription omitted the word "tick" after "deer" in 2-3 of the choices.
watch your transcriptions, people!]

(A) Whether populations of the other species on which deer tick larvae feed are found only in the areas also inhabited by white-footed mice.
(B) Whether the size of the deer tick population is currently limited by the availability of animals for the tick's larval stage to feed on.
(C) Whether the infected deer tick population could be controlled by increasing the number of animals that prey on white-footed mice.
(D) Whether deer ticks that were not infected as larvae can become infected as adults by feeding on deer on which infected deer ticks have fed.
(E) Whether the other species on which deer tick larvae feed harbor any other bacteria that ticks transmit to humans.
RonPurewal
ManhattanGMAT Staff

Posts: 16635
Joined: Tue Aug 14, 2007 8:23 am

Re: RC - Lyme Disease - GmatPrep


Sun Aug 30, 2009 3:55 am

do you have a specific question about this problem? if you just post a problem, it's hard to tell exactly what you're asking.

are there two or three answers between/among which you're deliberating? is there something you don't understand about the correct answer?

please post back, with your question(s). thanks.

note:
takeaway:
this question prompt
Which of the following it would be most useful to ascertain in evaluating the argument?
is very closely related to FIND THE ASSUMPTION.
if you see this prompt, you should think about the same sorts of issues that you'd consider in a Find the Assumption question.

--

here's another page about the same question. they must be cutting their question writing budget.
post2001.html
Pueden hacerle preguntas a Ron en castellano
Potete fare domande a Ron in italiano
On peut poser des questions Ron en franais
Voit esitt kysymyksi Ron:lle mys suomeksi

Un bon vtement, c'est un passeport pour le bonheur.


Yves Saint-Laurent
montz1
Forum Guests
Posts: 2
Joined: Sat Sep 05, 2009 8:12 pm

Re: RC - Lyme Disease - GmatPrep


Mon Sep 07, 2009 11:40 am

RonPurewal wrote:do you have a specific question about this problem? if you just post a problem, it's hard to tell exactly what you're asking.

are there two or three answers between/among which you're deliberating? is there something you don't understand about the correct answer?

please post back, with your question(s). thanks.

note:
takeaway:
this question prompt
Which of the following it would be most useful to ascertain in evaluating the argument?
is very closely related to FIND THE ASSUMPTION.
if you see this prompt, you should think about the same sorts of issues that you'd consider in a Find the Assumption question.

--

here's another page about the same question. they must be cutting their question writing budget.
post2001.html

For the question posted by gagansb...

I picked D because it helps to establish whether there is an alternate way through which ticks can acquire the bacterium and thus we can strengthen or
weaken the argument.
My reasoning:
Increasing the population of other species that do not harbor the bacterium may help in decreasing the number of ticks that acquire the bacterium - but
only when the ticks are in the larvae stage, if deer ticks that were not infected as larvae can become infected as adults, then they do acquire the
bacterium and hence the argument is weakened.

OA is B. I am not clear why the size of the deer population is relevant.


The deer population may be limited by the availability of animals on which ticks feed on in larvae stage, but if all these animals are white-footed mice
(or any other species that harbor the bacterium) the ticks would acquire the bacterium and in that case the entire deer population is infected. But if all
these animals are the 'other species' then ticks would not acquire the bacterium.
So, only the availability of animals does not help in drawing any conclusion, we need to know whether the animals are white-footed mice or the other
species.

Can you please explain what is wrong with my reasoning and why B is the correct answer?
RonPurewal
ManhattanGMAT Staff

Posts: 16635
Joined: Tue Aug 14, 2007 8:23 am

Re: RC - Lyme Disease - GmatPrep


Wed Sep 30, 2009 5:22 am

I picked D because it helps to establish whether there is an alternate way through which ticks can acquire the bacterium and thus we can strengthen or
weaken the argument.
...
My reasoning:
Increasing the population of other species that do not harbor the bacterium may help in decreasing the number of ticks that acquire the bacterium - but
only when the ticks are in the larvae stage, if deer ticks that were not infected as larvae can become infected as adults, then they do acquire the
bacterium and hence the argument is weakened.

nah. you're thinking too hard.

the conclusion of the argument is:


if the population of these other species were increased, the number of ticks acquiring the bacterium would likely decline.

you are mistakenly thinking that ANYTHING that would increase the # of infected ticks is irrelevant to this argument.

you have to stick to this conclusion, though. anything that does not have to do with the CONNECTION between INCREASING THE
POPULATION OF OTHER HOST SPECIES and the # of infected ticks ... is irrelevant.

analogy:
let's say the conclusion of an argument is if we institute a gmat prep program at State University, the students' gmat scores will go up.

if i make a statement such as students at State U have recently been getting higher gmat scores because they're studying on their own, this has NO
BEARING AT ALL on the conclusion above.
it has to do with increasing gmat scores, but not with the CONNECTION between instituting a prep program and increasing those scores.

OA is B. I am not clear why the size of the deer population is relevant.


The deer population may be limited by the availability of animals on which ticks feed on in larvae stage, but if all these animals are white-footed mice
(or any other species that harbor the bacterium) the ticks would acquire the bacterium and in that case the entire deer population is infected. But if all
these animals are the 'other species' then ticks would not acquire the bacterium.
So, only the availability of animals does not help in drawing any conclusion, we need to know whether the animals are white-footed mice or the other
species.

Can you please explain what is wrong with my reasoning and why B is the correct answer?

heh.

your problem here is that you were looking at the wrong words. someone mistranscribed the problem.
see edits above. (b) should say "deer tick population", not just "deer population".

hope that makes more sense now.


Pueden hacerle preguntas a Ron en castellano
Potete fare domande a Ron in italiano
On peut poser des questions Ron en franais
Voit esitt kysymyksi Ron:lle mys suomeksi

Un bon vtement, c'est un passeport pour le bonheur.


Yves Saint-Laurent
acethegmat
Students

Posts: 32
Joined: Wed Nov 19, 2008 5:42 am

Re: RC - Lyme Disease - GmatPrep


Sun Oct 25, 2009 1:52 am

[RON: this is my post; i'm not sure why it's credited to another user.]

I am unable to understand how B addresses the problem.


Even though the number of larvae increases, the number of infected larvae would be less, which is related to the conclusion.

the reason why you don't see how (b) addresses the problem is because you're making the same questionable assumption that the argument makes! in
fact, here you are making the precise assumption that is called into question by choice (b).

in particular, you are saying "the number of infected larvae would be less" -- this involves an assumption that the population of deer ticks will diffuse
and spread out.

choice (b) presents the possibility that the deer tick population is currently limited by the availability of hosts. if that's the case, then increasing the
number of hosts will NOT cause the population to dissipate or spread out -- instead, you'd just get exponential population growth, with
the same density of deer ticks on disease-causing hosts (as well as more of them on your newly introduced hosts).

D addresses the issue stating that whether the larvae feed increases or not, it will not help if the bacterium is caused through the deer itself.

nope, (d) is irrelevant because it has nothing to do with the substance of the actual argument, which deals with the result of introducing additional host
species for the larval tick.

you are making the mistake of thinking that any factor that affects the infected deer tick population, in any way, is relevant to this argument.
that's not true; the only things that are relevant are those that directly have to do with whether increasing the number of larval hostswill increase the
infected population. if you pick a choice that affects the infected population in some other way that doesn't have anything to do with larvae, then that
choice is irrelevant.

if you don't see what i'm saying, then here is an analogy that's almost certainly easier to understand:

argument:
studies have shown that increased protein intake promotes weight gain. therefore, if i increase my protein intake by eating egg whites every
morning for breakfast, i will be successful in gaining weight.
* whether eating egg whites will cause a feeling of satiation that will make me eat less protein throughout the rest of the day --> this is relevant,
because it actually deals directly with the effect of egg whites on my protein intake.
* whether there are other sources of protein that will be better than egg whites for achieving my goal --> irrelevant, since the passage isn't about
meeting my goal in general; the passage is only about whether egg whites, in particular, will help me meet that goal.

No?

You might also like